You are on page 1of 186

ENGINEERING: PHYSICS 1A

Course Lecturer: RUTH WABWILE

Introduction

These lecture notes are for 1ST years Engineering students (Electrical, Civil, Mechanical, Geospacial,
petroleum and Environmental & Biosystems) for their first semester Physics course. The notes cover
mechanics and properties of matter. The course is intended to introduce the student to the science
behind physical entities that will be covered in various areas of engineering. Please note that these
notes cover only PHYSICS 1A, the first semester engineering physics course, PHYSICS 1B will be
covered in second semester. The section on vectors is not covered here but the student can get the
background on vectors elsewhere.

COURSE OUTLINE

SECTION I: MECHANICS AND PROPERTIES OF MATTER


1. Motion in one and two dimensions
Kinematics: Vector & scalar quantities.
Resolution and composition of vectors.
The inclined plane.
Displacement.
Velocity.
Acceleration.
Equations of linear motion.
Motion under gravity.
Projectiles.
Newton’s laws of motion.
Linear momentum.
Principle of conservation of linear momentum.
Elastic and inelastic collision.
Impulse.
Circular motion: Angular displacement, Angular momentum, the radian measure, Angular velocity,
Period, Frequency, Acceleration, Centripetal force, Vertical and horizontal circular paths.
Static equilibrium.
Moments.
Couples. Torque. Work. Energy power

Page 1 of 186
2. Rotational dynamics
Rotation of rigid bodies.
Equations of motion.
Moment of inertia

3. Simple harmonic motion


Definition.
Relation to circular motion.
Velocity.
Acceleration.
Period.
Frequency.
Kinetic energy.
Potential energy.

4. Gravitation
Newton’s laws of Gravitation.
Kepler’s laws

5. Properties of Matter
Hooke’s law.
Stress.
Strain.
Young Modulus.
Coefficient of friction.
Pressure, Pascal principle.
Archimedes principle.
Coefficient of viscosity.
Stoke’s law.
Bernoulli’s principle.

SECTION II: SOUND AND VIBRATIONS


6. Introduction to sound
Wave phenomenon, general wave equation, sound waves, velocity of sound wave, interference,
beats and beat frequency, and Doppler Effect.

SECTION III: THERMAL PHYSICS


7. Heat
Internal energy and temperature, phase changes of pure substance, isothermal and isobaric
compressibility of gases, liquids and solids.
Page 2 of 186
8. Heat transfer
Conduction, convection and radiation.
Kinetic theory of gases, perfect gas equation, intermolecular forces, specific heats and
equipartition of energy.

Reference Books
1. Ohanian Physics II Edition, Norton & Co., NY (1989)
2. Vector Analysis, Schaum Series by Murray, R., McGraw Hill, NY (1980)
3. Theoretical Mechanics, Schaum Series by Murray, R., McGraw Hill, NY (1986)
4. College Physics, 3rd Ed, by Miller, Harcourt B. Inc. NY (1972)
5. A-Level Physics 5th Ed. by Nelkon & Parker, Heinemann (K) Ltd (1991)
6. Any book that covers mechanics, heat and sound
etc.

Page 3 of 186
SECTION I
MECHANICS AND PROPERTIES OF MATTER
MECHANICS is the branch of physics that deals with the action of forces on bodies and with motion,
comprised of kinetics (studies forces that cause changes of motion), statics (studies objects that are
either at rest, or in constant motion), and kinematics (is the area of physics that studies types of
motion without specifying what caused it or study of motion of objects).

PROPERTIES OF MATTER include any traits of an object that can be observed and be measured.
Everything around us has mass, volume and they occupy space, thus called matter.

Plasma

Four states of matter

This topic will be covered in Part 5: Hooke’s law, Stress, Strain, Young’s modulus, Coefficient of
friction, Pressure, Pascal principle, Archimede’s principle, Coefficient of viscosity, Stoke’s law, and
Bernoulli’s principle.

Page 4 of 186
CHAPTER 1: MOTION IN ONE AND TWO DIMENSIONS

1.1 LINEAR MOTION (MOTION IN A STRAIGHT LINE)


The mathematical concept of vectors is very useful for the description of displacement, velocity and
acceleration in one, two or three dimensions. A body can undergo either one of the following types of
motion or a combination of two or more of these motions namely:

i.) Translational or rectilinear motion i.e. motion in a straight line.

ii.) Rotational or circular motion e.g. a rotating wheel or planets around the sun

iii.) Vibrational or oscillatory motion e.g. a pendulum clock, atoms or electrons in a metal or solid.

Page 5 of 186
KEY TERMS USED IN LIEAR MOTION
a) Displacement (S ) is a physical quantity that specifies the position of an object (e.g. at point B )
relative to the initial point /origin e.g. point A in the figure below or it is the distance moved
(e.g. AB ) in a given direction.

So Displacement = shortest distance, and Distance = Total path covered!

b) Velocity (v) is vector that specifies the rate of change of displacement with time.

S
It is defined by: v (1)
t

Change in Displacement
The average velocity of an object is given by: v
Time Taken

Page 6 of 186
For example, consider the displacement-time graph given below:

S
B

P2
S2

P1

S1

t
A t1 t2

If S 2  S1 is the change occurring in position in time interval t 2  t1 , then the average velocity is

S2 - S1  S
v   slope of the straight line (2)
t2 - t1 t

On the other hand, the speed (v) between t1 and t 2 is defined by:

Actual Distance Curved Distance P1P2 S


v  v , as t  0
t2  t1 t t

i.e. speed is the rate of change of distance with time ( and in some cases is equal to the magnitude of
the velocity).

Total Distance
Average speed 
time taken

The instantaneous velocity is the quantity that tells us how fast an object is moving anywhere along
its path (e.g. A to B in the graph above). It is the average velocity between two points on the path in
the limit that the time (and therefore the displacement) between the two points approaches zero.

Page 7 of 186
Mathematically: Let position S be a continuous function of t denoted by S  t  ,
S (t2 )  S (t1 )
So average velocity between the two points is v  (2)
t2  t1

To find the instantaneous velocity at any position, we let t1  t and t2  t  t  as t  0 :


The equation for average velocity becomes:

S (t  t ) - S (t ) S (t  t ) - S (t ) dS  t 
 v(t)  lim  lim 
t 0  t  t  - t t 0 t dt

The instantaneous velocity of an object is the limit of the average velocity as the elapsed time
approaches zero, or the derivative of S with respect to t :

d
 v(t)  S t  (3)
dt

TAKE NOTE: The zeros of the velocity function give the minimum and maximum of the position
function, see Figure below.

d
Instantaneous speed = the magnitude of the instantaneous velocity:  v(t )  S t 
dt

c) Acceleration (a ) is a vector specifying how fast the velocity of a body changes with time i.e.

Change in Velocity  
a= = 
Time taken t t (3)

Page 8 of 186
v f - vi v
Average acceleration  a   
t f - ti t

d
The instantaneous acceleration is defined by: a inst 
dt

TAKE NOTE: Average acceleration approaches instantaneous acceleration as Δt approaches zero.


See the Figures below:

Their Units are: Displacement (m), Speed (ms-1), Velocity (ms-1) and Acceleration (ms-2).

The summary in graph form:

Page 9 of 186
EXAMPLE 1

Page 10 of 186
TYPES OF LINEAR MOTION
The three major types of simple linear motion are constant/velocity motion, uniformly accelerated
linear motion, and free fall.

(i) Motion with uniform velocity

Velocity-time graphs and displacement-time graphs are valuable ways of depicting motion in a
straight line. The figures below shows the displacement- and velocity-time graphs for a body moving
with uniform velocity i.e. constant speed in a fixed direction. The slope of Fig (a) gives the velocity.

v
S

t t
(a) Displacement-time graph (b) Velocity-time graph

For the displacement-time graph, the slope gives the velocity of the object while the area has no
physical significance. In the velocity-time graph, the slope is the acceleration of the object while the
area is the displacement of the object.

(ii) Motion with uniform acceleration (It derives the three types of Equations of motion/ laws of
constant acceleration)

The linear equations of motions] derivation can be done in three ways which are:
 Derivation of equations of motion by Simple Algebraic Method
 Derivation of Motion by Graphical Method
 Derivation of Motion by Calculus Method

Using algebraic method to derive the linear equations of motion:

If the velocity of a uniformly accelerating object increases from a value u to v in time t , then from
the definition of acceleration that

v -u
a  rate of change of velocity = (4)
t

Page 11 of 186
we have v  u  at (1st Equation of Motion) (5)

For uniform motion, the displacement covered in time t is defined by:

v  u 
S  Average velocity  Time =  t (6)
 2 

Substitute the v in Equation 6 with the v of Equation 5:


  u  at   u  2ut  at 2
S  t 
 2  2

1 2
 S  ut  at (2nd Equation of Motion) (7)
2

Using Equation 6 and substituting its t with the t of Equation 5:


v  u   v  u   v u  1
S    t        uv  u 2  v 2  vu 
 2   2   a  2a
Eqn.6 Eqn.5

 2aS = v 2 - u 2

 v2 =u 2  2as (3rd Equation of motion) (8)

TAKE NOTE: Equations 5, 7 and 8 are equations of motion for an object moving in a straight
line with uniform acceleration.

Page 12 of 186
Example A body covers a distance of 10m in 4s, it rests for 10s and finally covers a distance of 90m in
6s. Calculate its average speed.

Solution Total distance  10m  90m  0m  100m

Total time  4s 10s  6s  20s

100m
Average speed   5ms 1
20s

Example A student runs 800m due north in 110s followed by 400m due south in 90s. Calculate his
average speed and his average velocity for the whole journey.

Solution
800m

400m

Total distance 800  400 1200m


(i) Average speed     6ms 1
Total time 110  90 200 s

Total Displacement 800  400


(ii) Average velocity    2ms 1
Total time 110  90
Page 13 of 186
For velocity, since it is a vector, you have to choose the direction (+ or -).

Example A car moving with a velocity of 54kmhr-1 accelerates uniformly at the rate of 2ms-2. Calculate
the distance traveled from the place where the acceleration began to that where the velocity reaches
72kmhr-1 and the time taken to cover this distance.

Solution Given 54km.hr 1  15m.s 1.(u ) 72km.hr 1  20m.s 1.( ) a  2m.s -2 ,


then

(i)  2  u 2  2aS   20m.s 1   1 5m.s 1   2(2m.s 2 ) S


2 2
 S  43.75m

(ii)   u  at  20m.s 1  15m.s 1   2m.s 2  t  t  2.5s

(iii) Motion under gravity (free falling bodies)

A freely falling body is an object moving freely under the influence of gravity only, regardless of its
initial motion (also known as projectile motion). Objects thrown upward or downward and those
released from rest are all falling freely once they are released. Projectiles travel in trajectories/smooth
curved paths that take the shape of a parabola, see the figure below.

If the only force acting on the object is gravity as shown in the above figure, then the object will keep
the same horizontal velocity (ux) throughout its trajectory (Newton's First Law). The object will start
Page 14 of 186
to slow down in its vertical velocity (uy) the instant it is released (thus deaccelerates). When the object
reaches its peak it has a vertical velocity of 0m/s. The object then starts to increase in velocity on its
decent (thus accelerates).

The diagram below shows the trajectory of a projectile (in red), the path of a projectile released from rest with
no horizontal velocity (in blue) and the path of the same object when gravity is turned off (in green).The
position of the object at 1-second intervals is shown. The initial horizontal velocity is 20 m/s and there is no
initial vertical velocity ( i.e., a case of a horizontally launched projectile) and no horizontal
acceleration.

A body released near the earth’s surface will accelerate towards the earth under the influence of
gravity. If air resistance is neglected, then the body will be in free fall and the motion will proceed
with uniform acceleration of a  g  9.81m.s 2 . The value of this downward acceleration ( g ) is the
same for all bodies released at the same location and is independent of the bodies’ speed, mass, size
and shape.

Page 15 of 186
The equations of motion for freely falling bodies are similar to those for linear motion with constant
acceleration a being replaced by g . For upward motion (rising body), g is negative since the body is
decelerating. Thus

  u  gt
1 2
S  ut  gt (9)
2
 2  u 2  2 gS

Example A ball is thrown vertically upwards with a velocity of 20 ms-1. Neglecting air resistance,
calculate
(i) The maximum height reached, and

(ii) The time taken to return to the ground.

Solution Taking upward direction as positive, u  20ms 1 and a   g  10ms 2 then

(i) At maximum height,   0ms 1 thus

 2  u 2  2aS  0  20  20  2(10) S
S  20m
(ii) On return to ground, S becomes zero, thus from

1 1
S  ut  at 2  ut  gt 2
2 2
1
 0  ut  gt 2   20ms 1  t   5ms 2  t 2
2
  20ms 1  t   5ms 2  t 2

t  4s

Page 16 of 186
1.2 MOTION IN TWO DIMENSIONS (PROJECTILE MOTION)

Projectile motion is when an object moves both vertically and horizontally on the surface of the Earth
at the same time. The two motions are independent of each other i.e. the object moves horizontally
with constant speed, and at the same time, it moves vertically in a way a similar object not
undergoing horizontal motion would move. If air resistance is neglected, then a projectile can be
considered as a freely falling object and its equations of motion can be determined from the linear
equations of motion together with the initial conditions i.e. the initial velocity has components
u cos along the horizontal- and u sin along the vertical direction. The horizontal and vertical
motions are analyzed as follows:

deacceleration motion
acceleration motion

Where u is the initial velocity (u0), uy is the component of u along the y-axis and ux is the component
of u along the x-axis.
Note that at the origin O: t  0 seconds, u0  u, u0 y  u0 sin  , and u0 x  u0 cos .
Because projectiles move differently in x- and y- directions, there are two separate sets of equations
for modelling projectile motion (one setfor the x-axis and another et for the y-axis).

Page 17 of 186
The projectile never speeds up or slows down in the x-direction, thus ux is constant,
u x 0
 ax    0 . With x0  0 and ax  0 :
t t

1 2
x  axis : x  u0 x t  gt , but its g  0  x  u0 xt ; u x  u0 x  gt  u0 x (a)
2

For y-direction with y0  0 and a y   g :

1 2
y  axis : y  u0 y t  gt ; u y  u0 y  gt (b)
2

Which are equations for motion with constant acceleration ( a  g ).

PROJECTILE MOTION FORMULA/TRAJECTORY FORMULA


It is used to find distance, velocity and time taken in the projectile motion. Thus equations for time of
flight (t), maximum height reached (H) and horizontal range (R) are derived.

VERTICAL MOTION:
To determine the total time of flight (t) and maximum height (H).

(i) Total time of flight (t):


The resultant displacement (h) = 0 in Vertical direction when an object moves from O to B.
The vertical component of u y is  u sin   and the acceleration is  a y  – g  .
So use some trigonometry to resolve this vector U into its components (x- and y – components) as
shown in the figure above:
U U
cos   x  U x  U cos  ; sin   y  U y  U sin 
U U
When the projectile reaches the ground at B, the vertical distance (h) traveled is zero [h+(-h)=0].
So from constant acceleration equations of motion (the 2nd equation of motion):
1 1
s  ut  at 2  h  u y t  gt 2 (1)
2 2

we have,
1 2 u sin 
0   u sin   t  gt  t  2 (2)
2 g
which is the total time of flight (t).

Page 18 of 186
TAKE NOTE
Also at the maximum height (H) of the path, the vertical velocity (uy) of the projectile is zero.
So applying the 1st Equation of motion  v  u  at  in a vertical direction, the time (t) to
reach A is given by u y  u0 y  at :

 0   u0 sin    gt peak  0   u sin    gt peak

u sin 
 t peak   this is just half the time to reach B, see Eqn. 2   3
g

u sin 90o u
 t peak  
g g

(ii) Maximum height (H) reached at point A:


The vertical component of the velocity Uy will be zero.
So H can be evaluated as follows, using the 3rd Equation of motion:

v 2  u 2  2a s  u y 2  uoy 2  2 gH

 0   u0 sin    2 gH   u sin    2 gH
2 2

u 2 sin 2 
H  (4)
2g

HORIZONTAL MOTION:
(iii) To determine the range ( R ).
Since g acts vertically, it has no component in the horizontal direction ( ax  g  0 ).
So the projectile moves in a horizontal direction with a constant velocity u x   u cos   , which is
the horizontal component of u as derived above.
 
From the equations of motion s  ut we have the range (R):

r r  2u sin   2 sin  cos  sin 2


s  ut  R  ux  t   u cos       2u  u2 (5)
 g  g g

Page 19 of 186
TAKE NOTE
But maximum range (Rmax) is obtained when  sin 2   1 or 2  90o .

In this case:

u2
Rmax  (6)
g

Example An object is thrown horizontally with a velocity of 10m/s from the top of a 20m-high building
as shown. Where does the object strike the ground?

10ms-1
Solution: UX=U=10m/s, ax=0, H=20m, R=?
20m We consider the horizontal (to determine R) and
vertical (to determine t) problems separately.

(i) In the vertical case if the downward direction is taken to be positive, then u y  voy  0ms 1 since
is at H, g  9.81ms 2 and H  y  20m . Thus we can find the time taken to reach the ground from
1 1
2nd Equation of motion: H  u y  a y t 2  20m  0ms 1    9.81ms 2  t 2 from which
2 2
t = 2.02s.

(ii) In the horizontal case, we have found that the object will be in the air for 2.02s. Therefore given
that ux  u  10ms 1 and t  2.02s , then R  uxt  10ms 1  2.02s  20.2m .

1.3 NEWTON’S LAWS OF MOTION


Isaac Newton (January 4, 1643 to March 31, 1727) was a physicist and mathematician who developed
the principles of modern physics, including the laws of motion, and is credited as one of the great
minds of the 17th century Scientific Revolution. Newton made discoveries in optics, motion and
mathematics.

First Law: A body tends to remain at rest or in uniform motion in a straight line (with constant
velocity) unless acted upon by a resultant force. The tendency of a body to continue in its initial state
of motion (a state of rest or a state of uniform velocity) is called inertia. Accordingly, the first law is
often called the law of inertia.

Page 20 of 186
Second Law: If a net force acts on a body, it will cause an acceleration of that body. That acceleration
is in the direction of the net force and it magnitude is proportional to the magnitude of the net force
F
and inversely proportional to the mass of the body ie a  so that F  ma .
m
From the definition of a Newton, the law can be written in the form: F  kma  ma
This (vector) equation is a relation between vector quantities F and a , and is equivalent to the three
algebraic equations:
Fx  max
Fy  ma y
Fz  maz

Page 21 of 186
Third Law: Action and reaction are always equal and opposite, i.e. when one body exerts a force on
another, the second exerts an equal, oppositely directed force on the first.

Examples include:
 when pushing on a car, the car pushes back against your hand, when a weight is supported by
a rope, the rope pulls down on the hand (see the figure below);
 a book resting on a table pushes down on the table, and the table in turn pushes up against
the book; and
 the earth pulls on the moon holding it in a nearly circular orbit and the moon pulls on the
earth causing tides.

Page 22 of 186
The law differs from the first and second in that, whereas the first and second laws are concerned
with the behavior of a single body, the third law involves two separate bodies. The inherent
symmetry of the action-reaction couple precludes/prevents identifying one as action and the other
as reaction.

1.4 COLLISIONS AND LINEAR MOMENTUM


r
Linear Momentum is defined as the product of the object’s mass (m) and its velocity v , and is a
vector.

 
Linear momentum P  mass  velocity  mv

The SI unit of linear momentum is kg.m.s-1(Newton second-Ns=(kg⋅m⋅s−2).s) and its dimension

is
M L  .
T 

From Newton’s second law ( F  ma ) , if no external force acts on an object, then

 v u  mv  mu
F  ma  m  F 
 t  t

 Ft  mv  mu  Ft  P

P
F   0, if P is a constant.
t
Page 23 of 186
Thus its linear momentum is conserved. This is the principle of conservation of linear momentum. It is
useful in solving problems involving collisions between bodies.

The product of the force and the time is called the impulse of the force ( I ), i.e.
F  ma  m  v / t 

but Impulse  I   Fdt

 m(dv / dt )dt
 mdv

= mv  u

 mv  mu
 dP

 I  Ft  mv  mu  change in momentum)

The SI unit of impulse is the same as that of momentum i.e. Newton-second or kilogram-meter-per-
second.

COLLISION
Is any strong interaction between bodies that lasts a relatively short time.
Examples include: automobile accidents, neutrons hitting atomic nuclei in a nuclear reactor, balls
colliding, the impact of a meteor ("shooting star") on the surface of earth, a close encounter of a
spacecraft with the planet Saturn etc.
In all collisions, the linear momentum and the total energy are conserved. However, kinetic energy
might not be conserved since it might be converted into other forms of energy like sound, heat or
work during plastic deformation. There are two main types of collisions: elastic and inelastic collisions.

Elastic collision: both kinetic energy and linear momentum are conserved.

ua ub va vb
ma mb ma mb
ub vb
ma ua mb va ma mb

(i) before collision (ii) after collision

From the above figures, we have:

Page 24 of 186
maua  mbub  ma va  mb vb conservation of linear momentum

1 1 1 1
ma ua 2  mbub 2  ma v a 2  mb vb 2 conservation of kinetic energy.
2 2 2 2

Inelastic collision: linear momentum is conserved but kinetic energy is not conserved. If the colliding
r
bodies stick together (they will move with same velocity ( V )), they have undergone a perfectly
inelastic collision and hence we have:

ua ub V
ma mb ma mb vb
ub
ma ua mb ma mb

(i) before collision (ii) after collision

maua  mbub  (ma  mb )V conservation of linear momentum

SPECIAL CASES:
(i). Elastic collision in a straight line
Also known as Head-on collision or One dimensional collision or Direct collision. It means the colliding
bodies moving in one dimension before and after the collision.
If the two objects A and B have equal masses (m) and mass B is stationary ( uB=0) then for the elastic
collision we have:

mA mB
mA mB
mA vA mB vB
uA A B
mAA BB
m uB  0 va vb
ua
ub = 0
(i) before collision (ii) after collision

mu A  mv A  mvB conservation of linear momentum (i)

 uA  v A  vB (ii)

1 1 1
and mu A 2  mv A 2  mv B 2 conservation of kinetic energy (iii)
2 2 2

 u A2  v A2  v B 2 (iv)

Page 25 of 186
Rearrange Equation (ii) (  v A  u A - v B ) and substitute it in Equation (iv):

 u A2   u A - v B   v B 2
2

 uA = vB (v)

Thus the two objects simply exchange velocities (as shown by Equation (v) and in the figure below),
i.e. mass A comes to rest while mass B moves off with the original velocity of mass A. This is a
situation of maximum energy transfer between two colliding bodies and is mostly applicable in
nuclear reactions where neutrons are stopped by the protons.

(ii). Oblique collisions of equal masses


Oblique/angled collisions: Non Head-on collision or multi-dimensional collision (the two colliding
bodies are moving in two dimensions before and after the collision). That is it occurs when the
centers of mass of two bodies are not along the same straight line; after collision bodies are deflected
at some angle with the initial direction.

NOTE: There are two type of oblique collisions:


(i) Elastic oblique collision (both the kinetic energy and the linear momentum of the colliding
bodies remain conserved) and
(ii) Inelastic oblique collision (linear momentum of the colliding bodies conserved but the
kinetic energy is not conserved).

Here we will discuss the elastic oblique collision of two bodies with equal masses.
If mass A collides obliquely with mass B which is at rest and both objects are of equal
masses mA  mB  m , then the total momentum of any object will be the sum of the respective
momentum components in the vertical and horizontal directions respectively: P  Px  Py .
For any collision of two particles, this result implies that the momentum in each of the directions x, y,
and z is constant. In this type of oblique collision momentum conservation is applied separately along
x-axis and y-axis. If the collision is perfectly elastic, energy conservation is also applied.

Page 26 of 186
vy

vx
- vy

Where angles A   , B   , v x   v Ax  v Bx  and v y   v Ay  v By  .

Using a vector diagram for the momenta of the two balls before and after the collision, conservation
of linear momentum ( mu A  mv A  mv B ) gives:

Along the x-axis: u A  u, uB  0 ,

 muA  muB  mv Ax  mv Bx

 mu  mv A cos   mvB cos  along the x -direction ……………..……. (i)

Along the y-axis: u A  uB  0 ,

 mu A  muB  mv Ay  mv By

 0  mv A sin   mvB sin  along the  y direction ………………. (ii)

Conservation of kinetic energy gives:


1 1 1
mu A 2  mv A 2  mv B 2  u A 2  v A 2  v B 2 ………………….(iii)
2 2 2

Example

Page 27 of 186
Show that the two equal masses undergo oblique elastic collision will move at right angles after
collision, if the second body is initially at rest.

Solution
Let mass of each body be m=m1=m2,   1 ,   2 (assuming you are using above sketch) , initial
velocities are u = uA and uB = 0 respectively.

According to conservation of linear momentum:

mu  mv1cos1  mv2cos2 [along horizontal direction ] ............................................(1)

0  mv1sin1  mv2 sin2 ………………………………………………………………............ (2)

For an elastic collision, kinetic energy is also conserved:

1 1 1
mu 2  mv12  mv 2 2
2 2 2

 u 2  v12  v22 ……………………………………………………………………………………………........... (3)

Squaring and adding equations (1) and (2):

 mu   mv1cos1  mv 2cos 2 
2

2

+
0   mv1sin1  mv 2 sin 2 
2 2

u  v12  v 2 2  2v1v 2 (cos1cos 2  sin1sin 2 )


2

 u 2  v12  v22  2v1v2 (cos1cos2  sin1sin2 )

 u 2  v12  v2 2  2v1v2cos 1  2  ……………………………………………………………………….. (4)

Note that Equation (3) = Equation (4):

 u 2  v12  v22  0

 u 2  v12  v2 2  2v1v2cos 1  2 

Page 28 of 186
We confirm that:
0  2v1v 2cos 1  2  , since v1 and v 2 none is zero.


 cos 1   2   0  1   2   cos 1 0  rad  90o
2

(iii). Recoil
Recoil is a collision in reverse. It is a case where part of a composite body suddenly flies apart e.g. a
bullet fired from a gun, the remaining part (the gun) must undergo linear momentum in the opposite
direction (recoil) in order to conserve the linear momentum. See the figure below.

If mb and v b are the mass and velocity of the bullet while m g is the mass of the gun, then the gun will
recoil with a velocity v g given by,

Initial linear momentum = final linear momentum:

  mg vg  mb vb    mg vrecoil  mb vb  ……………………………………………….... (i)

From the above figure initially the gun and bullet are at rest, so their linear momentum is
zero:

 0   mb vb  mg vrecoil  ……………………………………………………………….………… (ii)

Comparing Equations (i) and (ii):

  mg vg  mb v b   0  mb v b  mg vg

or

Page 29 of 186
mb v b
vg   , it is far much less than v b since mb is much less than mg .
mg

Example A car traveling at 90kmhr-1 slams into a tree and is stopped in 40ms. If the car has a mass of
800kg, calculate the average force acting on the car during the collision.

Solution
u (initial velocity) = 90 km/h; t = 40ms; mass = 800kg; F = ?

v  90kmhr 1  25ms 1

From Ft  mv  mu ,

we have: 0.04 sec F  800kg  25ms 1  F  5  105 N

Example A person of mass 50kg who is jumping from a height of 5m will land on the ground with a
velocity v  2 gh  2  10  5  10ms 1 for g  10ms 2 .
1
If he does not flex his knees on landing, he will be brought to rest very quickly, say th second.
10
momentum 50 10
The force F acting is then given by F   5000N .
t 0.1
This is a force of about 10 times the person’s weight (500N) and the large force has a severe effect on
the body.

Suppose, however, that the person flexes his knees and is brought to rest much more slowly on
landing, say 1 second.
momentum 50 10
Then the force F now acting is 10 times less than before, or 500N: F   500 N .
t 1
Consequently, much less damage is done to the person on landing.

Page 30 of 186
1.5 CIRCULAR MOTION

Angular velocity

We consider the kind of motion where an object moves around a circular path about some fixed
point.

O is angular displacement

θ s θ s 2πr 2πr 2π
ω   ; v  r  rω;         t   T   
t t t v v ωr ω

⇒ , and this ratio is a constant for a given angle, another


method to show see the Figure above.

s
If , then  1 radian or 1rad.
r
If s is the full circumference of the circle, s/r = 2πrad, so 2πrad corresponds to 360° (so
1rad is about 57°).
Examples are: moon and earth revolving around the sun, the rim of a bicycle wheel, a stone being
whirled on a string etc.

In this chapter the system is assumed to move in a circle with a uniform speed around a fixed point O
as the centre. If the object moves from A to B so that the radius OA moves through an angle θ, its
angular velocity, ω, about O may be defined as the change of the angle per second. Considering time t

taken by the object to move from A to B we have   [units: radians per second].
t

The period (T) for this kind of motion is given by

Page 31 of 186
2
T since 2π radians is the angle in one revolution.

Derivation of the small angle approximations:

(S h, when )

If s is the length of arc AB, then s/r = θ, or s = r θ, taking

s 
 r  v  r (Relationship between angular and linear velocity).
t t

Angular acceleration (α)


An object moving in a circle at constant speed experiences a force that pulls it towards the centre of
the circular path. This force is known as centripetal force. If v is the uniform speed in the circle of
v2
radius r, then the acceleration is given by  . See the figure below.
r

Page 32 of 186
Linear/tangential, , refers to changes in the magnitude of velocity but not its direction
Centripetal acceleration, , refers to changes in the direction of the velocity but not its magnitude

HOW TO DERIVE ANGULAR ACCELERATION (  )

 v2 v2 (r )2 
There are two ways for determining  :    ; but v  r      r 2  :
 r r r 

v v v  v  s 
i)  , converting it into angular acceleration:     
t t t t r

v.  v t  v 2  r 
2

     r 2
t r r r
Page 33 of 186
OR

ii) Pulling out the trangle of angle  from the above figure:

    v 2      v 2 
sin 
 2    sin   = (1)
 v  t  t 
 2 v
  

r
If  is very small: v(t )  v  t   t   v , Equation (1) becomes:

    v 2 
 sin   =
 2 v
 

v   
 =sin   v (2)
2  2
 

For small angle   sin  , so Equation (2) can be rewritten as


 v   
 = v   v=  v
 2 
(3)
2  

 v   
 =   v (4)
 t   t 

 a =  v  a=  v  a =    r    2 r (5)

Page 34 of 186
CENTRIPETAL FORCES
Centripetal force is "the force that is necessary to keep an object moving in a curved path and that is
directed inward toward the center of rotation," it is actual force. While centrifugal force is "the
apparent force that is felt by an object moving in a curved path that acts outwardly away from the
center of rotation," according to Merriam Webster Dictionary.

So consider the situation below that shows an object of mass m whirled with constant speed v in a
vertical circle of centre O by a string of length r.

mg

T1
T2
B O

T3

mg C

mg

Let T1 be the tension in the string at point A (the highest point). Then since the weight mg acts
downwards towards O.

mv 2
Centripetal force (Fc) = mass  centripetal acceleration  Fc 
r
The force towards the centre is given by:

mv2 mv2
Fc  T1  mg   T1   mg ………………………………. 1
r r

Suppose T2 is the tension when the object is at point B, then at this point mg acts vertically
downwards and has no effect on T2. So,
Page 35 of 186
mv 2
F  T2  …………………………………………………………………………………… 2
r

Finally considering at point C where it is at the lowest point, mg acts in the opposite direction to T3
giving
mv 2
F  T3  mg  , leading to
r

mv 2
T3   mg ……………………………………………………………………………………………………. 3
r

Comparing T1, T2 and T3 from the above equations, it is seen that maximum tension in the string is at
point C (i.e, T3 is the highest). This implies that T3 must be greater than mg by mv2/r to make the
object keep moving in a circular path.

BANKING OF A ROAD
Is the phenomenon in which the outer edges of the curved roads are raised slightly above the inner
edge to provide the required centripetal force to the vehicles so that they take a safe turn without
slipping away through the roads or to give the driver a privilege not to lean more to take a turn.

Purpose of banking the road:


1) To contribute in providing neccesary centripetal force. This is the minimum force required to
let an object move along a curved path. It pulls an object toward the center of a circle as it
travels, causing angular motion or circular motion. The turn or the change of direction in which
the vehicle inclines towards inside is known as the banked turn.
2) To reduce frictional wear and tear of tyres.
3) To avoid skidding.
4) To avoid overturning of vehicles.

Suppose a car is moving round a banked road in a circular path of horizontal radius/radius of the
round r as shown in the Figure below.

Page 36 of 186
Below is a vector diagram for various forces, where r - horizontal radius/radius of the round and C -
center of gravity:

R = R1+R2

R2

R1
θ
mv 2 Inclined road surface
Fc 
r θ
B
F2

F=F1+F2 A
F1
θ
C Horizontal surface
r

mg

Deriving the angle of Banking/Banked angle (  ):


Let us consider a situation where friction force is absent, i.e., F1=F2=0, or the coefficient of friction is
zero.

Page 37 of 186
For horizontal equilibrium:
Fc  component of R in x  axis direction
If the only forces at the wheels A, B are the normal reactions R 1 and R2 respectively, then the force
towards the centre of the track (Fc) is (R1 + R2) sinθ where θ is the angle of inclination of the plane to
the horizontal. Hence,

Fc Fc
sin    sin     R1  R2  sin   Fc
R  R1  R2 

mv2
  R1  R2  sin   1
r

For vertical equilibrium:


When resultant friction force (F)=0

mg  component of R in y  axis direction :

mg mg
cos    cos  
R  R1  R2 

  R1  R2  cos   mg 2

Divide Equation 1 with Equation 2:

mv 2
 R1  R2  sin   sin  mv 2 sin  mv 2
 r    
 R1  R2  cos   mg cos  rmg cos  rmg

v2  v2 
 tan      tan 1   3
rg  rg 

Implying that for a given velocity  v  and radius  r  , the angle of inclination of the track for no sided
slip must be tan 1  v 2 / rg  . This has also been applied on rail tracks.

Page 38 of 186
Factors affecting angle of banking:
1) The angle of banking of a turn depends upon the velocity with which the vehicle is moving, and
if the velocity is doubled the value in the numerator will be four times and hence will increase
the angle. It is one of the major factors on which the angle of banking depends because the
angle is directly proportional to the square of the velocity.
2) The angle of banking also depends upon the radius of the curved path, it is inversely
proportional to the radius of the path.
3) It also depends upon the acceleration due to gravity, it is inversely proportional to
acceleration due to gravity. Since the value of gravity at the poles of the earth is more (10
m/s2), therefore the value of the angle of banking will be less at the poles as compared to the
quarter for the same speed of the vehicle.
4) The angle of banking is independent of the mass of the vehicle, therefore the mass of the
vehicle has no effect on the banking angle.

 
The safe velocity from Eqn.(3): vmax  rg tan  on a banked road depends on:
1) The radius of the curved path: The velocity is directly proportional to the radius of the curved path.

2) The acceleration due to gravity: Velocity is also directly proportional to the acceleration due to
gravity.

Higher the value of the gravity and radius higher can be the safe velocity. But also, the velocity is
independent of the mass of the vehicle, therefore the mass of the vehicle has no effect on velocity.

ANGULAR MOMENTUM  L 
It is the quantity of rotation of a body, which is the product of its moment of inertia  I  and its

angular velocity   , or it is the rotational equivalent of linear momentum  P 

Derivations from other quantities: L  I  r  P


In SI base units: kg. m2 .s 1

Dimension: M L2T 1

Consider a single particle of mass m which at one instant of time has a momentum P and is at a
distance r from the origin of coordinates. Below is a figure for particle in three-dimensional space, the
position vector r locates a particle in the xy-plane with linear momentum P . The angular momentum
with respect to the origin is L  r  P , which is in the z-direction. The direction of L is given by the
right-hand rule, as shown.

Page 39 of 186
r
L is perpendicular to the xy-plane
r
L

The magnitude of the angular momentum  L  is:

L  r P  L  r P

r
But sin    r  r sin 
r

 L   r sin  P  rP sin

Where θ is the angle between P and r .

The direction of L is along the perpendicular to the plane defined by the vectors P and r . The
r
direction of the vector L along this perpendicular is specified by the right-hand rule.

i.e. L  r  P

But P  mv , therefore

L  r  mv  L  mr  v=m  r  v  (units: Kgm2s-1)

The angular momentum of a particle moving with constant velocity in the absence of force is
constant, i.e. the magnitude of L  rP sin is constant and the direction of L is also constant. This
situation describes a free particle and represents conservation of angular momentum.

Page 40 of 186
Consider a case of conservation of angular momentum, i.e. a case of a particle in a uniform circular
motion, such as a stone whirled along a circle at the end of a string (Fig. below)

L
L
z

r
y

The vector L is perpendicular to the plane of the circle. In this case, since the position vector r is
always perpendicular to the velocity vector v , the magnitude of the angular momentum vector is

L  rp sin  mrv

So the direction of the angular momentum vector is perpendicular   90o  to the plane of the circle.
As the particle moves around the circle, L remains constant in magnitude and direction.

Page 41 of 186
SUMMARY

Page 42 of 186
1.6 EQUILIBRIUM
For an object to be in equilibrium, it must be experiencing no acceleration.

Types of equilibrium:
a) translational equilibrium: is a state in which net force is equal to zero.
b) rotational equilibrium: is a state in which net torque is equal to zero.
c) static equilibrium: is the state in which a system is stable and at rest.
To achieve complete static equilibrium, a system must have both rotational equilibrium and
translational equilibrium.

There are two conditions for equilibrium:


1. the vector sum of forces on the body MUST be ZERO, and
2. the vector sum of torques (force  lever arm) on the body MUST be ZERO.

1st condition for Equilibrium


Much of Physics has to do with objects and systems which are at rest and remain at rest. This portion
of physics is called statics. It is of prime importance since the concepts which it involves permeate
most fields of physical sciences and engineering.
For instance, an object is in equilibrium if it is not accelerating i.e. no net force must act on it. That
does not mean that no forces may be applied to the body. If several forces act simultaneously,
equilibrium demands only that the net force i.e. the vector sum of the various forces vanish (be
equal zero). This is the first condition of static equilibrium which can quantitatively be written as

F
i
i 0 (1)

which is equivalent to three component equations:

F i
ix 0

F i
iy 0 (2)

F i
iz 0

For example when an object rests on a table, there are two forces acting on it, namely its weight and
the upward reaction force of the table on the object. Without the table, the object cannot remain at
rest but would drop under the pull of gravity.

If the body remains at rest, it is said to be in static equilibrium while when it is in steady motion in
straight line, it is said to be in dynamic equilibrium. See the figures below.

Page 43 of 186
Man in static equilibrium Car in dynamic equilibrium

Torque and the 2nd condition of Equilibrium


Torque (  ) is a ‘twisting force’, thus a deciding factor in a state of equilibrium. It is defined as the
product of force ( F ) and lever arm ( r ) or simply the turning effects of a force.
The figure below is of a TORQUE-STATIC EQUILIBRIUM (Two children on a seesaw: The system is in static
equilibrium, showing no acceleration in any direction).

 Fulcrum 

From above figure r1  r,1 , r2  r,2 , F1  w1 , and F2  w 2 .

Thus   r  F  rF sin   or   r  F  rF sin 

  r  F  rF sin  (3)

where the lever arm ( r ) is the length of a perpendicular distance dropped from the pivot to the line of
the force (the perpendicular lever arm ( r ) is the shortest distance from the pivot point to the line along
which F acts) and  is the angle of rotation (see Fig. e below too).

If all the lines a long which several forces lie intersect at the same point, then the forces are said to be
concurrent. The second condition for equilibrium involves the torque that is applied to the object. This

Page 44 of 186
condition can be stated as: the resultant of all the torques acting on the object must be to cause no
turning effect i.e. the clockwise torques must balance the counterclockwise torques:


n
n 0 (4)

which can quantitatively be written as: torque    Lever arm×Force  r  F  rF sin  or

torque    r×perpendicular Force  r  F  rF sin 

Thus   r  F  rF sin same as Equation 3 above.

In the equation above,  is given by the right hand rule for the advance of a screw rotated from the
direction of r towards that of F . (Remember the vector cross product!).

where r is the magnitude of the lever arm ( r ), F is the applied force and  is the angle of rotation
(see Fig. e below).

Page 45 of 186
TAKE NOTE:
 The higher the torque value the easier it is to move the load around its fulcrum.
 Torque can also be calculated using the y-component of applied F :

   F  r, but F  F sin     rF sin  , as shown in the sketch below.

r F//

F F

where r is the distance between the pivot and where F is applied (as shown in Figs. a, b, d and e
above).

 Torques can be classified as either clockwise or counterclockwise. By convention, we take


counterclockwise (CCW) torques as positive (Fig. a) and clockwise (CW) torques as negative (Fig.
b).





 Moreover, a force whose line passes through the pivot causes zero torque. This is a reflection of
the fact the lever arm ( r ) for such a force is zero (see Fig. f above).

We therefore conclude that an object will be in equilibrium if the following conditions are
satisfied:

F ni  0 and  n 0

Page 46 of 186
Example
A long rope is stretched between points A and B. At each end the rope is tied to a spring scale that
measures the force the rope exerts on the supports. Suppose the rope is pulled sideways at its
midpoint with a force of 400N producing a deflection such that the two segments make angles of 5 o
with the line AB. What is the reading of the spring scales?

Solution

2T1 sin  or
2

1 2
1 2

1 2

Since the body is in equilibrium, we have


X-components:

T2 cos5o  T1 cos5o  0 (1)

Y-components:

T1 sin 5  T2 sin 5  400 N  0


o o
(2)

From equation 1, T1  T2 , substitute the T1 in Equation 2:

Therefore
T2 sin 5  T2 sin 5  400 N  0
o o

 2T2 sin 5  400 N  0


o

400 N
 T2  o
 2295 N
2sin 5
T1  T2  2295 N

The tension in the rope and therefore the force registered on the spring scales is 2295N.
Page 47 of 186
A force of 400N applied perpendicular to the line AB caused a tension of nearly 2300N, more than five
times the applied force in magnitude! There is a practical lesson to be learned here.

Example
Calculate the net torque acting on the rod below.
F2  400 N

1.5m

3m
F1  200 N

Solution

F2  400 N
  1 
1.5m

  2 
3m
F1  200 N

 net   1   2
Considering the torque's direction:

  net   1    2 

  net   r1  F1    r2  F2    3m  200 N   1.5m  400 N 

=600 Nm  600 Nm  0 Nm  0

So the rod is in equilibrium or it cannot rotate clockwise (CW) or counterclockwise (CCW).

Page 48 of 186
FRICTIONAL FORCES
Friction is the force that opposes motion between any surfaces that are in contact.
Frictional forces play an important role in the application of Newton’s Laws. There are three major
categories of frictional forces:
(i) Viscous frictional forces
Also known as fluid frictional forces. A fluid is a substance that can flow and take the shape of its
container. The force occurs when objects move through gases and liquids. An example is the
frictional force the air exerts on a fast moving car or plane. The air exerts a retarding force on the
car as the car slides through the air. Another example: If you've ever tried to push your open hand
through the water in a tub or pool, then you've experienced fluid friction.

(ii) Rolling frictional forces


Arise as, for example tire rolls on pavement. This type of friction occurs primarily because the tire
deforms as the wheel rolls. Sliding of molecules against each other within the rubber causes
energy to be lost.

(iii) Sliding frictional forces


Are forces that two surfaces in contact exert on each other to oppose the sliding of one surface
over the other. We will be concerned with sliding frictional forces.

(iv) Static Friction


It acts on objects when they are resting on a surface.
For example, if you are walking on the road, there is static friction between your shoes and the
road each time you put down your foot. Without this static friction, your feet would slip out from
under you, making it difficult to walk.

Page 49 of 186
Laws of solid friction
Solid friction is the friction which exists between two surfaces that are not lubricated. The two
Surfaces can both be at rest or one of the surfaces is moving and the other surface is at rest. So laws
of solid friction (or laws of static and dynamic friction or law of friction) state that:
i. The first law of friction states that the amount of friction is proportional to the normal force
exerted between the surfaces ( f  N ). As long as surface is at rest.
As the normal force increases, the friction force also increases proportionally. This is why it's
harder to push heavy objects than light objects.
ii. The second law of friction states that the limiting friction force does NOT depend on the shape
and area of contact between the object and the surface.
The maximum value of static friction ( f s ) is achieved when the body is just on the verge of
motion. The friction at this point is also known as limiting friction. Limiting friction is always
slightly greater than the kinetic friction, and is given by:

f s  max   µs N

where, f s  max  - limiting friction, µs - coefficient of static friction, and


.
N - Normal reaction force

Thus the kinetic frictional force ( f k  µk N ) and the static frictional force ( f s  µs N ) do not
depend on the area of contact as long as the Normal Reaction force is the same.

Imagine if you turned a box so that less of it was touching the floor. Would that change the
normal force? No! The box still weighs the same, so the normal force doesn't change, even if
the area of contact does.
iii. The third law of friction states that, in addition to the normal force, the friction force also
depends on the type of surfaces in contact.
Example, pushing a heavy box across a carpeted floor, it is harder than when you push it across
a smooth floor. This is because the friction force between the box and the rough carpet is a lot
bigger than the friction force between the box and the smooth floor.
iv. If the two bodies in contact with each other are slipping over each other, or having relative
motion with respect to each other, the friction that is involved is kinetic friction and is given by

Page 50 of 186
f k  k N

where, f k - kinetic friction, µk - coefficient of kinetic friction, N - Normal reaction force.

v. The direction of this kinetic friction on a body is opposite to the direction of the relative
velocity of the body with respect to the other body.
Suppose body A is moving over body B in the positive x-direction, then the kinetic frictional
force on body A will act in the negative x-direction.
vi. If the bodies are not having relative motion with respect to each other, the friction between
them is static friction ( f s ) and is always less than or equal to the kinetic friction.
It is important to remember that the value of static friction is not constant, as the case is with
kinetic friction.
The value of static friction depends on how much force is applied to the body. Suppose the
maximum value of static friction (limiting friction) is 5N for a body kept on a certain surface. If
you apply a force of 2N on the body, the value of static friction would be 2N, and the body will
not move. The body will start moving when the applied force is greater than 5 N.

Consider the following simple experiment


If a book resting on the table is pushed lightly with a horizontal force F the book does not move.
Apparently, the table-top also pushes horizontally on the book with an equal and opposite force. The
frictional force f opposes the sliding motion of the book and it is always directed parallel to the
sliding surfaces. If the pushing is increased slowly, then when the pushing force reaches a certain
critical value f s  max  , the book suddenly begins to move. Afterwards to keep the book moving a
smaller frictional force f k is enough. This simple experiment shows that two frictional forces are
important: the maximum static frictional force f s  max  that must be overcome before the object can
start moving and the smaller kinetic frictional force f k that opposes the motion of the sliding object.
The major reason for this behavior (cause of friction) is that the surfaces in contact are far from
smooth. Their jagged points penetrate one another and cause the surfaces to resist sliding. Once
sliding has begun, however, the surfaces do not have time to “settle down” onto each other
completely. As a result, less force is required to keep them moving than to start their motion. See the
graph below.

Page 51 of 186
The graph below shows the differences between static friction (is the force that has to be overcome in
order to get something to move) and kinetic friction (is the retarding force between two objects in
contact that are moving against each other):

The frictional force depends on how forcefully the two surfaces are pushed together. This situation is
described by what is called the normal force FN (where normal means perpendicular). The normal
force is the perpendicular force that the supporting surface exerts on the surface that rests on it. The
situation is shown below.

Experiments show that the frictional forces f s and f k are often directly proportional to the normal
force. In equation form, we have f s  s FN and f k  k FN . The factors  s and  k are called the static
and kinetic (or dynamic) coefficients of friction, respectively. They vary widely depending on the
nature of the surfaces involved as well as the cleanliness and dryness of the surfaces.

Page 52 of 186
In summary normal force decides how much of force the body exerts on the ground. See figures
below.

The coefficient of static friction  s can also be found by placing the block A on the surface S and then
gently tilting S until A is on the point of slipping down the plane. Let  be the angle of inclination of
the plane to the horizontal.

R fs

A
S


mgsin θ

mg
mgcos


Figure: Determination of Coefficient of friction by inclined plane

fs
s  (1)
R

fs
The static frictional force f s : sin    f s  mg sin  (2)
mg

R
The normal reaction R : cos    R  mg cos  (3)
mg

Page 53 of 186
Substitute Equations (2) and (3) in (1):

fs mg sin 
 s    tan  (4)
R mg cos 

and hence  s can be found by measuring  .

Example: Consider the situation shown

40N

5 kg 37o
f

W P

The applied force due to the rope is 40N, and the block has a mass of 5 kg. If the block accelerates at
3.0m/s2, how large frictional force must be retarding its motion?

Solution A free-body diagram for this situation is as shown below:

P F1  40 N
F1,y  40 N  sin 37o  24 N
37o 24N
f
F1, x  40 N  cos 37o  32 N
24N
W

Using Newton’s 2nd Law:


 Fx  max ;  Fy  ma y , a y  0

F1, x  f  max ;  W=P  F1, y

No acceleration in the y -direction since the block accelerates along the x-direction only,
and so we are not concerned with the y -forces: F y  ma y  m  0  0 .

Page 54 of 186
We have for the x -direction:
F x  ma x (1)

F x  5.0kg  3.0ms 2  15.0 kgms 2 (2)

Fnet , x  F1, x  f (3)

But F1, x  40 N  cos 37o = 32 N

 Fnet , x  F1, x  f  32 N  f (4)

Equation (2) = Equation (4):

15.0kgms2  32N  f

 f  32 N  15.0 kgms 2   32  15.0  kgms 2  17 kgms 2  17 N

Can you show that the coefficient of friction is 0.68 in this case?

f   N   P, but W  P  F1, y

 f   W  F1, y 

f 17kgms 2 17
    0.68
W  F1, y   5kg  9.8ms    40kgms sin 37  24.93
2 2 o

Example
A 20.0-kg crate is at rest on a floor as shown below. The coefficient of static friction between the crate
and floor is 0.700 and the coefficient of kinetic friction is 0.600. A horizontal force P is applied to the
crate. Find the force of friction if

(a) P =20.0N,
(b) P =30.0N,
(c) P =120.0N, and
(d) P =180.0N.
(e) its acceleration

Page 55 of 186
Solution
Free-body diagram of the crate to indicate the forces on the crate. Let f represent either the static or
the kinetic frictional force (because we have not determined whether the crate is subject to static
friction or kinetic friction).

Applying Newton’s second law (F=ma) in the horizontal (x-axis) and vertical (y-axis) directions,
including the friction force in opposition to the direction of motion of the box:

Newton’s second law:

 Fx  max ;  Fy  ma y

P  f  max ; N  w  N -w 0

The weight of the crate is:

w  mg  (20.0kg )(9.80m / s 2 )  196 N


 N  196 N , because w  N

Maximum force of static friction ( f s  max   s N  (0.700)(196 N )  137 N )

As long as applied P  137 N , the force of static friction ( f s ) keeps the crate stationary, and f s  P .

(a) P  137 N  P  f s  20.0N  f s  f s  20.0N


(b) P  137 N  P  f s  30.0N  f s  f s  30.0N
(c) P  137 N  P  f s  120.0 N  f s  f s  120.0 N

(d) P137 N , so the crate moves and now static friction vanishes and kinetic friction takes up.

f k  k N  (0.600)(196 N )  118N

Page 56 of 186
P f
(e) the acceleration is: P  f  max  ax 
m

P  f k 180.0 N  118N 62kgm / s 2


 ax     3.10m / s 2
m 20.0kg 20.0kg

1.7 WORK, ENERGY AND POWER


Work and energy can be considered as two sides of the same coin. In this section, we will learn all
about the concept of work, energy, and power.

Work is said to be done when a force applied to an object moves that object/ the ability to change
energy, and it is measured in joules (J).
Energy is the capacity to do work, and it is measured in joules (J).
Power is the rate at which work is done i.e. energy converted/ is work done per unit time, and it is
measured in watts (W).

WORK
It is a force causing the movement/displacement of an object. For work to be done a force must be
exerted and there must be displacement in the direction of the force. Thus is the product of the
   
force F , the displacement S of the point at which the force is applied and the cosine of the angle

  between the force vector F and the displacement vector S /direction of motion. When F and S
are in different directions as shown in the figure below, only the component of F in the direction of
S matters (i.e. Fy  F  F sin  or Fx  F//  F cos ).

Fy  F
F

Fx  F//  F cos

Mathematically (using above figure):

W  component of force parallel to the direction of motion  displacement  F/ / .S

 W  F .S  FS cos   W  F .S  F S cos 


Page 57 of 186
where W is the amount of work done by the force of magnitude F during a small displacement of
magnitude S .

We can regard the product  S cos   as the component of the displacement S   in the direction of
the force  F  , or alternatively, regard the product  F cos  as the component of the force  F  in the

direction of the displacement  S  .

TAKE NOTE: Force and displacement are vectors while work is a scalar due to dot scalar product/dot
product rule.

Work done against gravity:

W  mgh
which is same as the system's potential energy (P.E or V)

Features of the definition


i. First work requires the action of a force. Without a force, no work is done.
ii. Second, the application of a force is a necessary but not sufficient condition for work.
iii. Work is done only if there is displacement of the point of application of the force, and then only if
this displacement has a component along the line of action of the force.
iv. Although work is the product of two vector quantities, it is a scalar.
v. Its SI unit is the (Newton-meter) or (kgm2s-2) and is given the name Joule. One joule is the amount
of work done by a force of one Newton acting over a distance of one meter in the direction of the
displacement 1J  1Nm  .

Page 58 of 186
Below are Figures for various examples of work:
a)

The work done by the force on


this lawn mower is W  Fd cos .

TAKE NOTE: F cos is the


component of the force in the
direction of motion.

b)
A person holding a briefcase does
no work on it, because there is no
displacement. No energy is
transferred to or from the
briefcase.

c)

The person moving the briefcase horizontally


at a constant speed does no work on it, and
transfers no energy to it.

TAKE NOTE: The work done by a force is zero


if the displacement is either zero or
perpendicular to the force.

d)

Work is done on the briefcase by carrying it


upstairs at constant speed, because there is
necessarily a component of force F in the
direction of the motion.
Energy is transferred to the briefcase and could
in turn be used to do work.

Page 59 of 186
e)
When the briefcase is lowered, energy is transferred
out of the briefcase and into an electric generator.
Here the work done on the briefcase by the generator
is negative, removing energy from the briefcase,
because and are in opposite directions.
W  F cos  Fd cos180o  Fd (1)   Fd .

TAKE NOTE: cos   cos180o  1

Example
Calculate the work done by a man of mass 65kg in climbing a weightless ladder 4m high. Where
g  10ms 2 .

Solution

Mass = 65kg, h = 4m, W=?

work done (W) = force  distance


h
= weight  distance = mgh

= 65kg  10ms-2  4m=2600 J =2.6 kJ


O A
mg

Example
How much work is done in lifting a 3kg mass a height of 2m and in lowering it to its initial position?
Where g  9.8ms 2 . (i) Since the force is directed up and the displacement is in the
Solution same direction during lifting,   0o .
 W  mgh  3kg  9.8ms 2  2m  58.8J
F F (ii) Suppose we now slowly lower this mass to its original position.
Again we must apply an upward force of mg to prevent it from
dropping. How much work is done by this force? Now the
h  2m angle between that force and the displacement is   180o and
since cos 180o   1 , we have
 W   mg cos   h  3kg  9.8ms 2  cos180o  2m  58.8 J
Page 60 of 186
The negative sign tells us that some other agent, gravity, has done work on the body.
In this example, there are two forces that act on the 3-kg mass: the force of gravity, which points
downwards and the tension in the arms which pulls upward.

TAKE NOTE: If we had asked for the work done by the force of gravity, it would have been negative
during the lifting of weight (due to  g ) and positive as the weight was lowered (due to  g ).

ENERGY
Energy is defined as the capacity to do work.

A system may have mechanical energy by virtue of its position, its internal structure or its motion.
There are also other forms of energy besides mechanical, namely chemical energy (found in foods,
oils, charcoal, biogas etc and is due to the kinetic energy and potential energy of the electrons within
atoms), electrical energy (associated with the electric charge and can be produced by generators from
hydroelectric power stations-waterfalls, geothermal stations, nuclear fission etc-), nuclear energy
from a nuclear reactor, thermal energy (due to heat produced from burning fuels, the sun, heaters
etc). See the forms of energy chart below.

Page 61 of 186
It is a remarkable fact about our physical universe that whenever one form of energy is lost by a
body/system, this energy never disappears but it is merely translated into other forms of energy, e.g.
Vehicles burn fuels to produce both thermal(heat) and mechanical energy. In this section we will look
at mechanical energy only.

MECHANICAL ENERGY
It is the energy of motion - whether that energy is in action (K.E) or stored (P.E). It exists in two forms:

(i) Kinetic energy ( T or K.E ) - energy possessed by a body by virtue of its motion, and it represents
the capacity of the body to do work by virtue of its speed.

If a force F acts on an object of mass m such that the mass accelerates uniformly from initial
velocity  vi or u  to a final velocity  v f or v  ms 1 over a distance S as shown below,

u v
m F m
F

S
Then the work done over the distance S is:
W  F .S  Wnet  Fnet .S (1)

Page 62 of 186
Since the mass accelerates uniformly/constant, then applied force is constant too and it is parallel to
the displacement S . Therefore the force must obey Newton’s 2nd law:

F  ma  Fnet  ma (2)

Substitute Equation 2 in Equation 1:

 Wnet  ma.S (3)

Using 3rd Equation of motion to determine the constant linear acceleration  a  :

v2  u 2
v 2  u 2  2aS a (4)
2S

Substitute Equation 4 in Equation 3:

 v2  u 2  1
 Wnet  m   .S  2 m  v  u  (5)
2 2

 2S 

That the work done is:

1
W  ma.S  m  v 2  u 2   K .E or T (6)
2

which is the WORK-ENERGY RELATION (THEOREM), which states that the net work on an object
causes a change in the kinetic energy of the object.

Example

a) Calculate the net work done on the above parcel.


b) Solve the same problem as in part (a) by finding the work done by each force that contributed
to the net force.

Page 63 of 186
Solution
a) Fnet  Fapp.  f

 Fnet  120 N  5.00 N  115N

Wnet  Fnet  d  115N  0.800m  92.0Nm  92.0J

b) Identify all the forces the parcel will experience by sketching as sown below:

W  F .S  FS cos  , where  is the angle between the force and direction of displacement .

Total work done by each force Wtotal   Wg  WN  Waap.  W f

 Wtotal   mg cos 90o  0.800m    N cos 90o  0.800m  

120 N cos 0o  0.800m    5.00 N cos180o  0.800m 

 Wtotal   0    0    96.0 J    4.00 J   92.0 J

Confirmed that Wtotal  Wnet , thus it doesn’t matter which method you use to determine the Wnet .

(ii) Potential energy  V or U  - energy possessed by a body by virtue of its configuration (position)
in a force field e.g. gravitational field, electrostatic field, magnetic field, etc.

If an object of mass m is lifted to a height h from the ground, then:

Work done on the mass W  F .h  mgh (7)

That is work done on the object  gain in the potential energy by the object.

Page 64 of 186
Whether a body falls vertically or slides down an inclined plane, the work done on it by gravity
depends only on its mass and on the difference in height between the initial and final positions.
Potential energy of an object depends only on its location and not on the route by which it arrived
at that point. It follows that if a body is transported around a closed path, the change in potential
energy vanishes i.e. potential energy is independent of the previous history because the
gravitational force is a conservative force. A force is said to be conservative if the work WAB 
done by the force in moving a body from A to B depends only on the position vectors rA and rB .
In particular, a conservative force must not depend on time, or on the velocity or acceleration of
the body, see the figure below.

TAKE NOTE:
Non-conservative work is a work (e.g. work done by friction) that depends on the path.
Conservative work (e.g. gravity potential) is path independent. Suppose I push a block along a
surface with friction from point A to point B along the two paths shown.

The work done along path 2 will be greater


than path 1. However, if this was work
done by gravity (no friction) then the work
done along the two paths would only
depend on the starting and ending points.
It turns out that for any conservative
forces (e.g. gravity, springs, electrostatic)
you could make that work a potential
instead of a "work done by".

Page 65 of 186
Example
A 100kg crate of milk is pushed up a frictionless 30 o inclined plane to a 1.5m-high platform. How much
work is done in the process? where g  9.8ms 2 .

Solution
N

d B

100kg
r h  1.5m
F  Fmg ,y  mg cos 

 Fmg , x  mgsin
A C
mg

  30o , m=100kg, g  9.8ms 2 , and h  1.5m, f  0 , d  ?, W  ?

Since the surface is frictionless, the work done to push the crate through path AB WAB  is equal to
the work done in lifting the crate through path CB WCB  . So there are two ways of calculating the
work done.

(a) WAB  F .d  F d cos  (i)

  0o

h 1.5m
sin   d   3m (ii)
d sin 30o

The x-component of mg is:

Fmg , x  mgsin30o (iii)

Fmg , x must be balanced by the applied force F to prevent the crate from slipping down the plane
since it is frictionless:

F  Fmg , x  mgsin30o  100kg  9.8ms 2  sin 30o  490kgms 2 (iv)

Page 66 of 186
Substitute Equations (iv) and (ii) in Equation (i):

1.5m
WAB  mgsin30o. o
 mg 1.5m   100kg  9.8ms 2 1.5m  1470kg.ms 2 .m  1470 J
sin 30

OR

WAB  490kgms 2  3m  1470kgm2 s 2 .m  1470 J

OR

(b)
WCB  mgh

 WCB  100kg  9.8ms 2 1.5m  1470kg.ms 2 .m  1470 J

Example
Suppose in the previous example the inclined plane is not frictionless and that the coefficient of
friction is 0.2. How much work is done in pushing the crate to the 1.5m-high platform?

Solution
N

d fk B

100kg
h  1.5m
F  mg cos 

 mgsin
A C
mg
  30o , m=100 kg, g  9.8ms 2 , and h  1.5m, f k  0.2 , d  ?, Wtotal  ?

W  F .d  Fd cos  , where  is the angle between the force and displacemend (d ) directions.

Page 67 of 186
Total work done/ net work:

   
Wtotal  WN ,  WN ,  WF  W f  N  , .d  N  , .d  F .d  f k .d

 Wtotal   mg cos  d cos   mg cos  d cos    mg sin  d cos   k mg cos  d cos 

=  mg cos30o d cos90o  mg cos30o d cos90o   mg sin 30o d cos 0o  0.2mg cos30o d cos180o

 Wtotal   0   mg sin 30o d cos 0o  0.2mg cos30o d cos180o (i)

and d is:

h 1.5m
sin   d   3m (ii)
d sin 30o

The x-component of mg is:

Fmg , x  mgsin30o (iii)

Fmg , x must be balanced by the applied force F to prevent the crate from slipping down the plane
since it is frictionless:

F  Fmg , x  mgsin30o  100kg  9.8ms 2  sin 30o  490kgms 2 (iv)

The force of friction is:

f k  k N  k mg cos30o  0.2(980 N )(0.866)  170 N (v)

Substitute Equations (ii), (iv) and (v) in Equation (i):

 Wtotal  mg sin 30o d cos 0o  0.2mg cos30o d cos180o

 1.5m 
1  0.2mg cos30o 
1.5m 
 Wtotal = mg sin 30o  o    1
 sin 30   sin 30o 

Wtotal = 1470 J  510 J  1980 J

Page 68 of 186
 Wtotal  pe  Wn  1470 J  510 J  1980 J (8)

The work done against the force of gravity is the same as before 1470J. However, the applied force (F)
must be greater than mgsin30o so as to overcome the force of friction which also acts in the negative
x- direction (opposite to the direction of motion).

TAKE NOTE: The work done against this frictional force is then:
 1.5m 
Wn   f k d  0.2(980N )(0.866)   510J
 sin 30 
where n indicates nonconservative forces.

Conservative forces are forces that store energy e.g. gravitation force, elastic (Hooke's Law) forces,
electric forces, etc. Like lifting a book, the work that you do "against gravity" in lifting is stored
(somewhere "in the gravitational field" or stored "in the Earth/book system") and is available for
kinetic energy of the book once you let go.

Nonconservative or dissipative forces are forces that do not store energy e.g. frictional force, etc. The
energy that it removes from the system is no longer available to the system for kinetic energy, like
pushing a book

POWER
Is the rate of doing work i.e. it is the rate at which energy is converted from one form to another.

Mathematically,

W
P (average power) (1)
t

W S
Also since W  F .S , then P  F  F .v  Force  velocity (2)
t t

The unit of power is the Watt (W) which is the rate of work (transfer of energy) of one joule per
second. Power is also measured in horsepower (hp), where 1hp=746W.

The efficiency  of a machine or system is the ratio of the power output to power input i.e.

power output
Efficiency     (3)
power input

Page 69 of 186
Example
A manually operated crane winch is used to lift a 200kg mass to the roof of a 10m tall building.
Assuming that you can work at a steady rate of 200W, how long will it take you to lift the object to the
roof? Neglect frictional forces.

Solution
Since f  0  W  F.d  F d cos   F d (cos 0o )  mgd .

The work done equals the increase in potential energy of the 200kg mass:

W  mgh  200kg  9.8ms 2  10m  19600J

Since this work is done at a constant rate of 200W, then

Work 19600J 19600J


Power   200W   t   98 sec
Time t 200W

Let us see how large an error may have been made by neglecting the kinetic energy of the mass
during the ascent. The average speed of the mass is:

10m
v  0.102ms 1
98sec

1 1
Kinetic energy during ascent is therefore: k .e  mv 2  (200kg )(0.102ms 1 )  1.04 J
2 2
an amount negligibly small compared with the change of 19600J in potential energy. We can
therefore safely neglect this small amount of kinetic energy in the problem’s solution.

TAKE NOTE
Since W  mgh which is equal to potential energy, then it implies that the neglected frictional
1
forces are the kinetic energy: E  T  V  E  T  mgh  E  T  W  mv 2  W
2

Page 70 of 186
Page 71 of 186
CHAPTER 2: ROTATIONAL DYNAMICS

2.1 Rigid Body


A rigid body is one that consists of millions of particles, each at different places. Remember the
angular momentum for a single particle

L  r  p  mr  v

where r is the position vector of the particle (relative to origin) and p is the linear momentum.

Total angular momentum ( L ) for a rigid body is the sum of the angular momentum of all particles in
the body. If these particles have masses mi, velocities vi and the position vector ri (relative to a given
origin of coordinates), then the total angular momentum is

n
L   mi ri  vi
i 1

where n is the total number of particles.

Note that the angular momentum of a rigid body obtained from the above formula depends on the
choice of origin of coordinates. See the Figure below.

Axis of rotation

where is the moment of inertia of a rotating rigid body about the axis of rotation.

Page 72 of 186
TAKE NOTE
When a rigid body rotates about a fixed axis, every particle in the body moves with the same
angular speed ω. The Figure below shows the link of the angular and linear quantities of a
rotating rigid body.

Reusultant/linear acceleration in
Vector form:  a   atan  arad , and Scalar form/magnitude: a  atan 2  arad 2

ds d  r  d
Linear velocity  v   v r  r
dt dt dt

dv d (r  ) d
Tangential acceleration of the rigid body  at    r  r
dt dt dt

v2  r 
2

Radial acceleration from circular motion  ar     r 2


r r

Moment of Inertia ( I )
Moment of inertia or angular mass or the mass moment of inertia or rotational inertia, of a rigid body
is a measure of an object's resistance to changes in its rotation rate. That is it is a quantity that
determines the torque needed for a desired angular acceleration   about a rotational axis; similar to
how mass determine the force needed for a desired acceleration. Its SI units is kg.m2. This is the
rotational analogue of mass in linear motion.

Consider a rigid body rotating about a fixed axis O, and a particle A of the object makes an angle θ
with a fixed line OZ in space at some instant.

Page 73 of 186
A

where K R is rotational kinetic energy.

The angular velocity ω of every particle is same everywhere. For a particle at A, the velocity is v1 = r1ω
(r1 = OA).

The total kinetic energy for the whole body is the sum of individual ke given by

   mi ri 2 
1 2
KR 
2

 1 
Comparing this with ke for linear motion  ke  mv2  shows that the magnitude of  mi ri 2 can be
 2 
denoted by the symbol I and is known as the moment of inertia of the object about its axis of
rotation. Correspondence    :
1 1
KR  I 2  ke  mv 2
2 2

I m
v
Factors that affect moment of inertia are the shape of the body, the mass of the body, and position of
the rotation axis.

Page 74 of 186
Torque on a rotating body

Consider a rigid body rotating about a fixed axis O.

The net force acting on a particle A (F1) = mass1 x acceleration1, given by:

dv1 d  d  d d  d 2 
F1  m1a1  m1   m1  (r1 )  m1  r1    m r
1 1  m1 1
r 
dt dt  dt  dt dt  dt 2 
 m1r1  

 F1  m1r1

The moment of this force/torque about the axis O = force x perpendicular distance from O, which is

1  F1  r1  r1F1 sin   r1F1 sin 90o  r1m1r1  m1r12

The total moment of all forces (or total torque) is

  m r   m r
1 1
2
2 2   ........  mn rn 2
2

 n 
     mi ri 2    I (it is analogous to  F  ma )
 i 1 

Correspondence    :     F ; ma  I  m  I and a  

Page 75 of 186
Conservation of Angular Momentum

The angular momentum of one particle is: L  r  P

  dP 
The rate of change of this momentum is:
dL d

dt dt
 
 dr
r  P    P   r 
 dt  

dt 

 dr 
Taking the first term on RHS of the above equation we have:   P   v  (mv)  m(v×v)  0 
 dt 

While the second term is equal to torque, where force F (force is equal to rate of change of linear
momentum ( P  mv )), this implies that
 dP 
 r    r 
dt 
d (m v)
dt
 r m
dv
dt

 r  ma  r  F 


dL
dt
 
 0 r F  r F 
dL
dt


For a rigid body, the total angular momentum is the sum of angular momentum of individual particles
and the rate of change of the total angular momentum is the sum of the rates of change of individual
angular momentum. That is

dL n dL n
  ri  Fi    i
dt i 1 dt i 1

where Fi is the force acting on the particle i.

But ri  Fi is the torque of the sum Fi on particle i .

dL n dL dL
   i     I
dt i 1 dt dt

If the forces acting on the particle are external, and if the total external forces are such that the total
external torque is zero, then the angular momentum is conserved, i.e.

dL dL

dt
 
dt
 0  dL  0   dL   0dt  L  constant

Page 76 of 186
This is the law of conservation of angular momentum, which states that when no external torque
   0 acts on an object, no change of angular momentum ( L  constant ) will occur.
SUMMARY

Page 77 of 186
Chapter 3: Simple Harmonic Motion (SHM)

3.1 Introduction

Simple Harmonic Motion is a special type of periodic motion/oscillation motion, where a restoring
force ( Fr ) is applied that is proportional to the displacement ( x ) and in the opposite direction of that
displacement [and the applied force ( F )]. A restoring force ( Fr ) is a force that tries to pull or push a
displaced object back to its equilibrium position. Whenever the system is displaced from equilibrium,
Fr urges the system to return.

Examples are: A pendulum swinging back and forth, the vibration of a guitar string, a mass vibrating at
the end of the spring, etc. All objects that vibrate have one thing in common: each system is subject
to a restoring force that increases with increasing distortion. See the figures below.

(a) (b) (c)

x
Fr

x Fr
x
Fr

The record of its vibratory motion, a displacement versus time graph is at least sinusoidal or
cosinusoidal form as shown below:

Page 78 of 186
There are certain terminologies used to describe vibratory systems: Crest, trough, wavelength,
amplitude, period, frequency, and wave velocity. We shall illustrate them by reference to the figure
below.

Crest — is the maximum value of upward displacement within a cycle.


Trough — is the opposite of a crest, so is the minimum or lowest point in a cycle.
Wavelength (λ) — is the distance between adjacent identical parts (i.e. crests or troughs) of a wave,
parallel to the direction of propagation.
Amplitude  A — maximum displacement from the equilibrium position of an object oscillating
around such equilibrium position. It is only half as large as the total vertical distance traveled by the
object.
Period T  — is the time it takes to complete one oscillation.
Frequency  f or   — number of events per unit of time (like number of waves passing by a specific
point per second):

1
f  1
T

The dimensions of frequency are (time)-1 or cycles/vibrations per second. One cycle per second is
denoted as one hertz (Hz) which is the SI unit of frequency.

Wave velocity/speed of propagation  v w  — is the speed at which the disturbance moves. That is
the distance the wave travels in a given time, which is one wavelength in a time of one period.

Page 79 of 186

vw  f
T

3.2 Simple Harmonic Motion


In engineering practice, we are almost invariably interested in predicting the response of a structure
or mechanical system to external forces; e.g. we may need to predict the response of a bridge or tall
building to wind loading, earthquakes, or ground vibrations due to traffic, etc. We will use a spring-
mass system as a model of a real engineering system.

Condition(s) a system must satisfy if its vibration is to be sinusoidal/basic conditions to execute


simple harmonic motion are:
1. There must be an elastic restoring force acting on the system.
2. The system must have inertia.
3. The acceleration of the system should be directly proportional to its displacement and is
always directed to mean position (i.e., the acceleration ( a ) being oppositely proportional to
the displacement ( X )  a   X ).

The velocity of the mass undergoing SHM can be derived using:


a) Newton’s second Law (F=ma), and
b) the law of conservation of energy (E=T+U)

Using Newton’s 2nd law to derive the velocity of the mass undergoing SHM
Consider the system shown below

Fr
m

Xo
O A

Simple harmonic motion occurs when the force ( Fr ) on an object is proportional and in the opposite
(-) direction to the displacement ( X o ) of the object (i.e. Fr   kX o ). If the spring obeys Hooke’s law
(which states that the force ( F ) needed to extend or compress a spring by some distance X scales
linearly with respect to that distance) then Fr   kX o . At equilibrium, the mass is at point O.
Suppose it is displaced a distance X o as shown and released. The restoring force Fr will pull the mass
back toward point O and the mass will vibrate around O as centre. If this motion is to be sinusoidal or
cosinusoidal in this case, then the displacement  X  of the mass will be given by

Page 80 of 186
2
X  X o cos t 2
T

TAKE NOTE
Recall: in uniform circular motion:
a) angular velocity   is constant, and the angular displacement   is related to the angular
velocity by the equation:   t

b) consider the y-component of the motion the path with time: y  t   A sin t  , which is
2
analogous to Equation 2: ( X  X o cos t )  X  X o cos t  X  X o cos 
T

Thus Simple Harmonic Motion, is a component of circular motion. Thus is defined as the projection
of uniform circular motion on any diameter of circle of reference. A particle spinning in circular
motion and another moving with SHM the r  A . See the Figure below for the interpretation of
above statement:

Page 81 of 186
2 2
The function [ cos t ] (in Equation 2: X  X o cos t ) is the oscillatory function traced as shown
T T
below in the graph of displacement against time.

Displacement  X 

Xo

Xo

Notice that cos goes through one complete cycle as θ goes from 0 to 2 (or 360o). In Equation 2
2
( X  X o cos t ), the angle goes from 0 to 2 as t goes from 0 to T . Hence T is the time taken for
T
one complete cycle and is the period.

Another feature about Equation 2 is the factor X o . Because cos oscillates between +1 (or 0o) and -1
(or 180o) as θ keeps increasing, the displacement X oscillates between + X o and - X o as time goes on.
Therefore X o is the amplitude of the vibration.

What sort of force acts on the mass to produce this sinusoidal motion?
It is simply the force Fr exerted on the mass by the spring. We can find Fr from our equation for X by
using Equation 2 to compute the acceleration of the mass and then by using F  ma to find Fr .
Newton's 2nd Law F  ma ; which is the force due to the motion of the body  F   Fr  ma  kx  .

To carry out this, we differentiate Equation 2 with respect to time in order to find the velocity of the
r
mass  v  . We have

dX d  2  2 2
v   X o cos t   X o sin t 3
dt dt  T  T T

Page 82 of 186
The velocity we have found is the velocity of the mass for the graph above. If we now differentiate v
v with respect to t , we obtain the acceleration of the mass  a  . It is

dv 2 d 2  4 2  2  4 2
a  X o  sin t   2  X o cos T t   2 X  4
dt T dt  T  T  T

TAKE NOTE:

 4 2 
1) a    2   X   a   X , it confirms that the acceleration of the mass is proportional to
T 
constant

the negative displacement.


2) It is important to remember that when using these Equations 2, 3, and 4, your calculator
must be in the radians mode.
3) Choose sine or cosine to derive the Equations ( X , v, and a ) of SHM. If the motion starts at
equilibrium position ( x = 0 or y  0 ), use the sine. If it starts at the maximum or minimum
position ( Ymax or X max or Ymin or X min ), use the cosine. Either will work.

Determining the displacement, velocity and acceleration of the mass when the projection is
taken along the x-axis or y-axis:

(a) x-axis: where ( ) and , where is angular velocity (   2 f ) . Then

Page 83 of 186
(b) y-axis, where ( ) and , then

 r 4 2 
This acceleration  Equation 4: a   2 X  is caused by the unbalanced restoring force Fr   kX  .
 T 
Therefore F  ma becomes

4 2
Fr  m X 5
T2

The restoring force is opposite in direction to the displacement X , a condition that is inherent in the
nature of restoring forces. In addition, the restoring force is proportional to the displacement and this
is equivalent to saying that the system obeys Hooke’s law. Thus we have arrived at the following
result: a system that vibrates sinusoidally obeys Hooke’s law in which the restoring force is
proportional to the distortion. Such a system obeys Equation 2 and is called simple harmonic motion.
Hence to test whether or not a vibratory system obeys a sinusoidal equation, we simply check to see
that the system obeys Hooke’s law.

From Hooke’s law, the sprig constant k is the proportionality constant so that

Fr  kX (Hooke’s law)

Page 84 of 186
This relation simply states that the restoring force is proportional to the distortion and is directed
opposite to the direction. If we compare this form of Hooke’s law and Equation 5, we see that

 4 2   m
k   2 m  or T   2  6
 T   k 

This is a very important relation because it gives the period of vibration of the mass m in terms of
the spring constant k . The period of vibration is long for large mass (or for large inertia) and for small
k (because the force exerted by the spring is small). Thus one characteristic of the SHM of an object
2
attached to a spring is that the angular frequency   (and therefore the period and frequency of
T
the motion), depend on only the mass (m) and the force/spring constant (k), and not on other factors
such as the amplitude of the motion.

The frequency of vibration ( f ) can also be derived as

1 1 1 k
f   
T  m  2 m
 2 
 k 

The velocity of the mass ( v ) can be derived as follows, substitute T in Equation 3 with Equation 6:

2 2 2 2
v X o sin t X o sin t
T T  m T
Eqn.2  2 
 k 

k 2
v X o sin t 7
m T

2 2
Square Equation 7 or Square Equation 3 ( v   X o sin t ) (take note of the trigonometric
T T
identities) to derive the linear velocity ( v ) below:
2
 k 2  k  2  k    2  
  v t    v    X o 2 sin 2 
2 2
  X o sin t     X o 2 1  cos 2  t  
 m T  m  T  m    T  

k 2 2  2  k
  v  t     X o2  X 2 
2
 X o  X o cos 
2
m  T  m

Page 85 of 186
k
v ( X o2  X 2 ) 8
m

Notice that v is maximum when X  O , that is, when the system passes through equilibrium.

TAKE NOTE
Notice that the velocity and acceleration are also sinusoidal, see the SHM time graphs below:

However, the velocity function has a 90° or (π/2) phase difference while the acceleration
function has a 180° or (π) phase difference relative to the displacement function.
For example, when the displacement is positive maximum, the velocity is zero and the
acceleration is negative minimum.

Example
When a 30g mass is hung from the end of a spring, the spring stretches 8.0cm. This same spring is
used in the experiment with a 200g mass at its end. The spring is stretched 5.0cm and released. If we
assume that the spring slides without friction, find the following for the mass:

(a) period of vibration,


(b) frequency,
(c) acceleration as a function of X, and the
(d) speed of the mass as it goes through the equilibrium position.

Page 86 of 186
Solution First we must find the spring constant. In a stretching experiment,
stretching force 0.30kg  9.8ms 2
k   3.7 Nm 1
stretch 0.080m

m 0.200kg
(a) T  2  2  1.466s
k 3.7 Nm 1

1
(b) f   0.682Hz
T

 3.7 Nm 1 
 X  18.5s  2 X
Fr kX
(c) a    
m m  0.200kg 

NOTE: Why is the acceleration greatest at the extreme of the vibration? Why is it zero when X=0?
k k
(d) When X=0, X o  0.05cm : v ( X o 2  X 2 ) vo  X o  0.215ms 1
m m

Using the law of conservation of mechanical energy (E=T+U) to derive the velocity of the mass
undergoing SHM
k
This is another method of deriving Equation 8: [ v  ( X o 2  X 2 ) ] above.
m
The Figure below shows the 5 critical points as the mass on a spring goes through a complete cycle
and the spring's Simple Harmonic Motion (SHM) long the y-axis - Position, velocity and acceleration
versus time.

Page 87 of 186
Below is the above spring’s Simple Harmonic Motion (SHM) – Position (x) along the y-axis, velocity (v)
and acceleration (a) versus time graph.

A B C D E

Y-axis
Start & back
to top (+A)

Equilibrium

Lowest point
(-A)

Page 88 of 186
Below is the transformation of mechanical energy in SHM for an object attached to a spring on a
frictionless surface. Where X is the position along the x-axis, Us is the potential energy stored in the
spring, K is the kinetic energy of the mass, v is the velocity and a the acceleration.
Let positions (b) and (d) be the equilibrium position (i.e. F r = 0) for the mass. The total energy stored in
1
the spring is kX o . Take note: Fr  F  Fr  ma  0  ma  a  0, m  0 .
2

-Xo 0 +Xo

v=0 Fr X Us K v a
(a) +Xo 1/2kXo2 0 0 -amax
-v
(b) Fr = 0 0 0 1/2m(-vmax)2 -vmax 0
Fr

(c) -Xo 1/2k(-Xo)2 0 0 amax

v
(d) Fr = 0 0 0 1/2m(vmax)2 vmax 0
v=0
(e) +Xo 1/2kXo2 0 0 -amax
Fr

TAKE NOTE: Oscillations About an Equilibrium Position 1) Kinetic energy is maximum at mean
position (i.e. when mass is at
equilibrium) and minimum at
extreme position (i.e. when mass is
at rest).
2) The mass is moving fastest when the
system is moving through its mean
position because then all the energy
is kinetic.
3) Potential energy is maximum at
extreme position and minimum at
mean position.
4) Total mechanical energy always
remains constant.

Page 89 of 186
When a spring is stretched a distance X o and the mass is at rest, the energy stored in the stretched
spring is all the potential energy. Thus

E  Us  K  E  Us  0

1
 Us  kX o 2
2

Because the mass is at rest at (a), kinetic energy K is zero. The restoring force (Fr) of the spring
accelerates the mass toward the left in (a) by the time the system reaches the configuration in (b), the
spring is no longer distorted, so U s  0 . The energy originally stored ( U s ) in the spring has been
changed to kinetic energy. Thus in (b) we have

1
 K  Us  kX o 2
2
1 1
Note: K  m   v max   mv max 2
2

2 2

Since K has now achieved its maximum value, the velocity of the mass is largest. But because Fr  0
in (b), the acceleration of the mass is zero (Recall: F  Fr  ma  Fr  ma  0. But m  0, so a  0 ).

In general:
1. The energy changes back and forth from U s to K . Thus at the both ends of the path, K  0
and U s is maximum and at the center, U s  0 and K is maximum.
2. The velocity of the mass is largest as the mass passes through the centre. It is zero when the
mass is at either end of the path.
3. The acceleration is zero at the midpoint and is maximum at the three ends (a, c, and e).

At any point along the X the total energy of the system is

1 2 1
Total energy  U s  K  kX  mv 2
2 2

1 2
Since the total mechanical energy stored in the spring is kX o , we arrive at a very important
2
relation stating how the energy of the system is apportioned/distributed:

1 1 1
kX o 2  kX 2  mv 2 9a
2 2 2
Page 90 of 186
Determining v :

1 1 1 m 2
kX o 2  kX 2  mv 2  X o2  X 2  v
2 2 2 k

m 2 k
 X o2  X 2  v  v2   X o2  X 2 
k m

k
v  X o2  X 2  9b
m

Equation 9b is the same as Equation 8.

TAKE NOTE
The motion of the block on a spring in SHM is defined by the position x  t   Acos t    with a
velocity of v  t    A sin t    . Using these equations, the trigonometric
k
identity cos 2  sin2  1 , and   , we can find the total energy of the system.
m

1 2 2 1
ETotal  kA cos t     mA2 2 sin 2 t   
2 2
1 2 2 1 k
 kA cos t     mA2   sin 2 t   
2 2 m
1 1
= kA2 cos 2 t     kA2 sin 2 t   
2 2
1 2 1 1
 kA cos 2 t     sin 2 t      kA2 cos 2  sin 2   kA2
2 2 2

The total mechanical energy of the system of a block and a spring is equal to the sum of the
potential energy stored in the spring plus the kinetic energy of the block and is proportional to
1
the square of the amplitude  ETotal  kA2 .
2
The total energy of the system is constant.
Another vibratory system of interest is the pendulum. The pendulum mass oscillates back and forth
between the positions shown below. At positions A and C, the energy is all potential U g   mgyo .
At any other position where the bob’s height is y , we have

Page 91 of 186
1
Total energy  mgyo   U g  K  mgy  mv 2
2

This is the basic energy equation for a pendulum. Other vibratory systems can be analyzed in a similar
way. In all of them, interchange between U and K occurs. The mass is moving fastest when the
system is moving through its equilibrium configuration because then all the energy is kinetic.

Example Suppose the pendulum below is released from a position where y o  20cm. Find the speed
of the mass at (a) point B and (b) when the value of y  1.30cm .

C
A
yo

B
Solution

yo  20cm, y  1.30cm

Using the energy method, we can write

1
mgyo  mgy  mv 2  v  2 g ( yo  y)
2

(a) At the mean position (B), velocity of the pendulum is maximum and displacement from the
ground is minimum ( y  0 ), therefore potential energy  mgy   0

Using y  0 gives v  2  9.8ms 2  (0.20  0)m  0.626ms 1

(b) In the same way with y  0.0130m we find v  0.37ms 1

The equation of motion for SHM


Simple harmonic motion occurs if the system obeys Hooke’s law where the restoring force is
proportional to the distortion. For a distortion X , this requires that

Fr  kX

where k is the spring constant and the negative sign is arising since it is a restoring force which is in
the opposite direction of the applied force. When X is positive, Fr will be directed toward – X . The
Page 92 of 186
equation of motion for a mass-spring system is obtained by writing F  ma for the system. Since
F  Fr in this case, we have

 kX  ma

However the acceleration itself depends on X because

dv d dX d 2 X
a   2
dt dt dt dt

Hence the equation of motion becomes

d2X k
2
 X 1
dt m

This is the typical equation of motion for SHM. Mathematicians call it a differential equation.

When you study differential equations, you will learn that in terms of periodic motion the solution of
this equation is

X  X o sin(2ft   o )

where the constant X o is the amplitude of the vibration and o is an arbitrary phase constant.

d2
X   X o  2 f  sin(2 ft  o )
2
a 2
dX 2

The angular frequency  f  and angular velocity   of a mass on a spring can be determined as
follows, as it relates to the spring constant and the mass:

  kX  ma   k  X o sin(2 ft   o )   m   X o  2 f  sin(2 ft   o ) 
2
 
  X o  2 f  sin(2 ft   o ) 
2

 k
 X o sin(2 ft   o ) 
m 
 o
X sin(2 ft   o 
)  o
X sin(2 ft   o 
)

  k   m  2 f 
2

k k 1 k 1
   2 f   2 f  f  
2
 
m m 2 m 2

Page 93 of 186
1 k 
 f   (in Hertz or revolutions per second) 3
2 m 2

2
   2 f  (in radians per second) 4
T

The result confirms that a system obeying Hooke’s law gives rise to sinusoidal vibration with the
frequency given as Equation 3 above.

Example
The spring-mass system shown below is vertical and is therefore influenced by the force of gravity.
Does it still undergo SHM?

Fr
y

mg
Solution
For SHM to occur: Fr  ky

But the net force acting on the mass is Fnet  Fr  mg   ky  mg

where down is taken as positive.

d2y
Using the F  Fnet  ma  Fnet   ky  mg   ma  ky  mg  m , then divide by k both sides
dt 2

ky mg ma mg ma  mg   ma 
    y   y    
k k k k k  k   k 

 mg   mg 
Let  y    y1 , then it indicates that   is simply the amount the spring stretches due to the
 k   k 
weight of the mass, since it is subtracted from y . By changing variables, we subtract this change in

Page 94 of 186
spring length. The quantity y1 is simply the displacement from the equilibrium position of the loaded
spring. With this new variable, the equation of motion becomes

 ma 
 y1      ma  ky1  F  ky1  Fr  ky1
 k 

d 2 y1
ma  ky1  m  ky1 which is the equation of motion for SHM
dt 2

And so the coordinate y1 undergoes simple harmonic motion with the frequency as though gravity
were not present. We therefore conclude that gravity shifts the equilibrium point but does not
otherwise affect the vibration.

Example
The Figure below shows the motion of a 45 gram ball attached to the end of a spring on a frictionless
surface. The spring has a spring constant, k , of 15 newton’s per meter.

a) What’s the period of oscillation?


b) How many bounces will you get per second?
c) Determine its angular frequency.
d) If the ball was pulled 10.0cm before releasing it, making an amplitude of 10.0cm. Find its
acceleration.

Solution
a)

b) The number of bounces represents the frequency:

Page 95 of 186
f  3Hz

c) The angular frequency is the angular velocity (  ). So,

d) At position A the mass is at its equilibrium position and it is where the motion starts ((see the
graph below, ignore magnitudes for t). So a sine wave is generated as it moves from A to B and
to C , then back to A .

So,

Displacement: 

Velocity: 

Acceleration: 

Page 96 of 186
Chapter 4: Gravitation
4.1 Introduction
The force that binds together progressively larger structures from star to galaxy to supercluster and
may be drawing them all towards the great attractor is the gravitational force (i.e. the weight of an
object (mg) is the gravitational force between the object and the Earth). This force not only holds you
on Earth but also reaches out across intergalactic space (which is the region of physical space that
exists between galaxies).

Physicists like to study seemingly unrelated phenomena to show that a relationship can be found if
they are examined closely enough. This search for unification has been going on for centuries. For
example, in 1665 Isaac Newton made a basic contribution to physics when he showed that the force
that holds the Moon in its orbit is the same force that makes an apple fall. We take this so much for
granted now that it is not easy for us to comprehend the ancient belief that the motions of
earthbound bodies and heavenly bodies were different in kind and were governed by different laws.

Newton concluded that not only does Earth attract an apple and the Moon but every body in the
universe attracts every other body: this tendency of bodies to move toward each other is called
gravitation. Newton’s conclusion takes a little getting used to, because the familiar attraction of Earth
for earthbound bodies is so great that it overwhelms the attraction that earthbound bodies have for
each other. For example, Earth attracts an apple with a force magnitude of about 0.8N. You also
attract a nearby apple (and it attracts you), but the force of attraction has less magnitude than the
weight of a speck of dust. Quantitatively, Newton proposed a force law that we call Newton’s law of
gravitation: Every particle attracts any other particle with a gravitational force whose magnitude is
given by

m1 m2
F G
r2

where m1 and m2 are the point masses of the particles, r is the distance between them, and G is the
gravitational constant, with a value that is given by G  6.67  1011 Nm 2 kg 2  6.67  1011 m 3 kg 1 s 2
(this value was determined by Henry Cavendish, a British).

Newton fitted well his New Universal law of gravitation above with his previous 3 Laws of
motion/Newton & calculus, and the 3 Kepler's Laws:

m1m2  M 
F G  mG 2   mg  F  ma
r 2
 r 
 M
Therefore gravitational acceleration  g    G a
 r2 

Page 97 of 186
As the Figure below shows, a particle m2 attracts a particle m1 with a gravitational force F that is
directed toward particle m2 and particle m1 attracts particle m2 with a gravitational force  F that is
directed toward m1 .

rEM

The forces  FME and FEM form a third law force pair; they are opposite in direction but equal in
magnitude. They depend on the separation of the two particles (e.g. earth and moon, as above), but
not their location: the particles could be in a deep cave or in deep space. Also the forces  FME and
FEM are not altered by the presence of other bodies (e.g. an apple in the above Figure), even if those
bodies lie between the two particles we are considering.

The strength of the gravitational force i.e. how strongly two particles with given masses at a given
separation attract each other, depends on the value of the gravitational constant G . If G , by some
miracle, were suddenly multiplied by a factor of 10, you would be crushed to the floor by earth’s
 
attraction ( FE you ). If G were divided by this factor, earth’s attraction FE you would be weak enough
that you could jump over a building.

Although Newton’s law of gravitation applies strictly to particles, we can also apply it to real objects as
long as the sizes of the objects are small compared to the distance between them. The moon and
earth are far enough apart so that to a good approximation, we can treat them both as particles. But
what about an apple and earth? From the point of view of the apple, the broad and level earth
stretching out to the horizon beneath the apple certainly does not look like a particle. Newton solved
the apple-earth problem by proving an important theorem called the shell theorem: A uniform
spherical shell of matter (e.g. earth) attracts a particle (e.g. apple) that is outside the shell as if all the
shell’s mass ( M   m1  m2  m3  ......  mn  ) were concentrated at its centre (e.g. center of earth).

Page 98 of 186
Newton's shell model states that:
1) A uniformly dense spherical shell of mass  M  will gravitationally attract an external mass
 m  - i.e. mass outside the shell, as if the mass of the shell were concentrated at its centre.
2) A uniformly dense spherical shell of mass  M  exerts no gravitational force on a mass  m 
located anywhere inside the shell. That is for the same sphere the gravitational field
inside the spherical shell is identically 0.
See the Figure below.

mb
G ( M 1  M 2  ....  M n )mb
Fb 
rb 2

rb

  M1  M 2  .......  M 3 
Exerts no force on m
GMm
Fa  ; when r  a
a2
Exerts force on m
GMm
Fr  ; when r  a
r2

GMmb
Fb 
rb 2

Earth can be thought of as a nest of such shells ( M1  M 2  .....  M n ) as shown above, one within
another and each shell attracting a particle outside earth’s surface as if the mass of that shell were at
the center of the shell ( e.g. M1  M ). Thus from the apple’s point of view, earth does behave like a
particle, one that is located at the centre of earth and has a mass equal to that of earth.

Page 99 of 186
TAKE NOTE:

Gravitating Spheres
Using Newton’s gravitational force, the distance between the mass m outside the earth and the
earth M was the radius of the earth (RE). In other words, we assume that all the mass of the earth
is concentrated at its center. This assumption may seem reasonable when we are far away from
the earth (that is we are at such a distance that the radius of the earth is negligible in
comparison), but it doesn't seem so good at all when we are at the earth's surface. However, we
will see that this assumption does hold exactly for any body outside the surface of a gravitating
sphere (to which the earth is a good approximation). This is a profound result. It is a consequence
of superposition, the inverse square law, and the symmetry of a sphere.

Example
Assuming the orbit of the earth about the sun to be circular with radius 1.5  1011 m , find the mass of
the sun.

Solution
R1  1.5 1011 m
The centripetal force needed to hold the earth in an orbit of radius R (note: the distance between the
centers of mass of Earth and an object on its surface is very nearly the same as the radius of Earth,
because Earth is so much larger than the object) is furnished by the gravitational attraction of the
sun: Centripetal force = Gravitational force

R1

Page 100 of 186


We therefore have, Centripetal force (Fc)  gravitational force (Fg)

mE v2 GmE mS R1v2
  mS 
R1 R12 G

where v is the speed of the earth in its orbit around the sun.
Since the earth rotates on its orbit once each year or in a time of  3.15 107 s  then the earth’s
circumference = the circumference of earth’s orbit around the sun, we have

Velocity = [circumference of the earth/period (time it takes to complete a circle)]

2 (1.5 1011 m)
v  3.0 104 ms 1
3.15 10 s
7

R v 2 1.5 1011 m  3.0 104 m / s 


2

 mS  1   2.0 1030 kg
G 6.67 1011 m3kg 1s 2

TAKE NOTE
Deriving the Earth's centripetal acceleration ( ac ):
Fg  Fc  GMm / R 2  mv 2 / R

 m   GM / R 2   mv 2 / R

 mg  mv 2 / R

 mac  mv 2 / R

 ac  v2 / R ------------------------------------------------------------------- (1), use it if v is known

 ac   R  / R  R 2 ----------------------------------------------------- (2), use it if the time (T) it


2

takes for the Earth to orbit around the sun is known

 
2

 ac  R   ------------------------------------------------------------------- (3)
t
 2 
2

 ac  R  R  2 f 
2
2
 R  -----------------------------------------(4)
 T 

Page 101 of 186


4.2 Gravitation near Earth’s Surface
Escape velocity
It is the minimum velocity needed to escape the gravitational field of a planet or other body.
If you fire a projectile upward, usually it will slow, stop and return to Earth. There is, however, a
certain minimum initial velocity that will cause it to move upward forever, theoretically coming to rest
only at infinity. This initial velocity is called the (Earth’s) escape velocity.

Consider a projectile of mass m leaving the surface of a planet or some other astronomical body or
system with escape velocity vesc , as shown in the Figure below.

R (M)

r  h  R

1 GMm
It has a kinetic energy K given by mv 2 and potential energy U given by U   .
2 R
In which M is the mass of the planet and R is its radius.

When the projectile reaches infinity, it stops and thus has no kinetic energy ( K f  0 ). It also has no
potential energy ( U f  0 ) because this is our zero-potential-energy configuration. Its total energy at
infinity is therefore zero ( E f  0 ).
From the principle of conservation of energy ( Ei  E f  U i  Ki   U f  Kf 
 U i  K i  0 (only at infinity) ), its total energy at the planet’s surface ( Ei ) must also have been
zero. So that

1 2  GMm  1  GMm 
K U  mv     0  mvesc  
2

2  R  2  R 

2GM  GM 
 vesc   2   2  v   v 2
2
R  R 
Page 102 of 186
where v is the orbital or magic velocity which occurs only when an escaping object's journey
eventually ends where it began around or completes an orbit, see Figure below).

Orbital velocity ( ):

But

2GM
The escape velocity ( vesc  ) does not depend on the direction in which a projectile is fired
R
from a planet. However, attaining that speed is easier if the projectile is fired in the direction the
launch site is moving as the planet rotates about its axis. For the rockets to escape from the Earth, you
need enough thrust to:
i. escape the gravitational pull of the Earth, and
ii. allow them to travel with very high speed i.e., escape velocity.

For example, rockets are launched eastward at Cape Canaveral to take advantage of the Cape’s
eastward speed of 1500km/h due to Earth’s rotation.

The equation above can be applied to find the escape velocity of a projectile from any astronomical
body provided we substitute the mass of the body for M and the radius of the body for R. the table
below shows some escape velocities from some astronomical bodies.

Body Mass (M) (kg) Radius (m) Escape speed (km/s)


Ceresa 1.17  10 21 3.8  105 0.64
Earth’s moon 7.36  10 22
1.74  10 6
2.38
Earth 5.98  10 24
6.37  10 6
11.20
Jupiter 1.90  10 27
7.15  10 7
59.50
Sun 1.99  10 30
6.96  10 8
618

Page 103 of 186


Sirius Bb 2  1030 1 107 5200
Neutron starc 2  10 30
1 10 4
2  105

a- the most massive of the asteroids


b- a white dwarf(a star in a final stage of evolution) that is a companion of the bright star Sirius
c- the collapsed core of a star that remains after that star has exploded in a supernova event.

4.3 Planets (in Greek meaning "wanderers") and Satellites: Kepler’s Laws
The motions of the planets as they move from East to west, as they seemingly wander against the
background of the stars, have been a puzzle since the dawn of history. The loop-the-loop motion/
retrogate motion of Mars was particularly baffling, it occurs after every 2 years (Earth moves very fast
than Mars). This motion is an optical illusion. See the Figure below (NOTE: when you are in your car
and overtaking another car on the highway, what is your observation on motion of the other car you
overtook?).

Motion along the


background of the
stars is from West to
East

West East

Watch these videos: https://www.youtube.com/watch?v=72FrZz_zJFU


https://www.youtube.com/watch?v=piBBQTkoQ9c

Johannes Kepler (1571-1630), after a lifetime of study, worked out the empirical laws that govern
these motions. Tycho Brahe (1546-1601), the last of the great astronomers to make observations
without the help of a telescope, compiled the extensive data from which Kepler was able to derive the
three laws of planetary motion that now bear his name. Later Newton 91642-1727) showed that his
law of gravitation leads to Kepler’s laws.

Page 104 of 186


In this section we discuss each of Kepler’s laws (Law of orbiting, Law of areas, and Law of periods) in
turn. Although here we apply the laws to planets orbiting the sun, they hold equally well for satellites,
either natural or artificial, orbiting Earth or any other massive central body.

Let us familiarize ourselves with terminologies used in KEPLER’S LAWS

Eccentricity of an ellipse.
All orbits are ellipses, i.e. basically stretched out circles. An orbit’s eccentricity (usually “e”) measures
how stretched out it is. The higher the eccentricity, the more stretched out the orbit and the more
offset the position of the star compared with the center of the ellipse. Thus eccentricity is a measure
of how closely a conic section resembles a circle.

When eccentricity =0 is a circle; an ellipse; is a parabola; is a hyperbola, and


for infinite eccentricity (e)=  is a straight line). See the Figure below.

Page 105 of 186


Mathematically using the Figure below, Eccentricity is the ratio of c to a; found by the following
formula: eccentricity = c/a, where c is the distance from the center to the focus of the ellipse and a is
the distance from the center to a vertex as shown in the Figure below whose foci are on the y-axis:

1. THE LAW OF ORBITS/KEPLER’S 1ST LAW


All planets move in elliptical orbits, with the sun at one focus.

The Figure below shows a planet of mass m moving in such an orbit around the sun, whose mass is M.
We assume that M  m so that the centre of mass of the Planet-Sun system is approximately at the
Page 106 of 186
centre of the Sun. The orbit is described by giving its semi-major axis (a) and its eccentricity
e   c / a    ea / a   e  , the latter defined so that ea is the distance from the centre of the ellipse to
either focus F or F’. An eccentricity of zero corresponds to a circle, in which the two foci merge to a
single central point. The eccentricities of the planetary orbits are not large so that the orbits look
circular. Example the eccentricity of the Earth’s orbit is only 0.0167.

F’

R p  (a  ea)  a(1  e); Ra  ea  ea  R p  ea  ea  (a  ea)  ea  a  a(e  1)

  R p  Ra   a(1  e)  a(e  1)  2a

and (r  r )  2a

2. THE LAW OF AREAS/ KEPLER’S 2ND LAW


It states that for equal intervals of time, the area covered by the earth/satellite will be same with
dA
respect to the center of mass of the sun/earth; that is the rate at which it sweeps out area A is
dt
dA
constant.   constant .
dt
Thus the angular momentum  L  R  P  R  mv  mRv  is conserved, where m is the mass of the
object, v is the orbit speed of the object, and R is the radius of the orbit. Since mass of the object (m)
does not change, when object is closer to the Sun (e.g. at perihelion), i.e. the radius R is smaller, the
orbit speed v must be greater, and vice versa. As a result, it sweeps out equal areas in equal time.

Page 107 of 186


t3
Perihelion Aphelion

t1

t2

Figure The shaded regions have equal areas ( A1  A2  A3 ).


It takes equal times for m to go from A to B, from C to D, and from E to F, i.e. t1  t2  t3 .

Qualitatively, this law tells us that the planet m will move most slowly when it is farthest from the sun
M and most rapidly when it is nearest to the sun.
As it turns out Kepler’s second law is totally equivalent to the law of conservation of angular
momentum as proved below.

Aphelion

Since A and t are constant as Kepler said in his 2nd law and m is constant too, then L must also be
constant-angular momentum.

Thus Kepler’s 2nd law is equivalent to the law of conservation of angular momentum.

Page 108 of 186


3. THE LAW OF PERIODS/Kepler’s 3rd Law
It states that the ratio of the squares of the periods of any two planets about the Sun is equal to the
ratio of the cubes of their average distances from the Sun.

T12 r13
 
T2 2 r23

This equation is valid only for comparing two small masses (e.g. m1 and m2 ) orbiting the same large
one ( e.g. M ). Most importantly, this is a descriptive equation only, giving no information as to the
cause of the equality.

Derivation of Kepler’s Third Law for Circular Orbits


Let r be the radius for the circular orbit (i.e. the radius of a circle is the semi-minor axis of an ellipse
and it is equivalent to the semi-major axis (a) of an ellipse ( r  a )) as shown in the Figure below.

r r

 sunr

The Newton’s laws of motion and his universal law of gravitation are used to derive the Kepler’s 3rd
law. The point is to demonstrate that the force of gravity  Fg  is the cause for Kepler’s laws
(although we will only derive the third law).

Since the planet (m) is orbiting around the sun (M), Gravity  Fg  supplies the centripetal force  Fc  to
mass m  i.e. Fg  Fc  , see Figure below.

Page 109 of 186


Starting with the Newton’s second law applied to circular motion:

v2
 net external force  Fnet   mac  m
r

The net external force  Fnet  on mass m is centripetal force  i.e. Fnet  Fc  , and so we substitute the
centripetal force for Fnet:

mv2 GM circular orbit's circumference 2 r


Fc  Fg  Fnet  Fg  m 2 , but v  
r r total time taken for a revolution T

m  2 r T 
2
 2 
2
Mm Mm
  G 2  m  r G 2
r r  T  r

4 2 M 4 2 M 4 2 T 2
  G   G  
T2 r3 T2 r3 GM r 3

 4 2  3
T2   r T  r T  a
2 3 2 3

 GM 

Now comparing two small masses (m1 and m2) orbiting the same large one (M) by taking the ratio of
the above Equation:

 4 2  3  4 2  3
T12   r
1 and T2
2
  GM  r2
 GM   

T12  4 2   GM  r13 r13 T12 r13 T12 a13


      
T2 2  GM   4 2  r23 r23 T22 r23 T22 ra3

The quantity in parentheses is a constant that depends only on the mass M of the central body about
which the planet orbits. The Equation holds also for elliptical orbits, provided we replace r with a , the
T2
semi major axis of the ellipse. This law predicts that the ratio 3 has essentially the same value for
a
every planetary orbit around a given massive body. The table below shows how well it holds for the
orbits of the planets of the solar system.

Page 110 of 186


Table: Kepler’s Law of Periods for the Solar System
T2
Planet semimajor axis a (1010m) period T (yrs) 3
(10 34 y 2 m 3 )
a
Mercury 5.79 0.241 2.99
Venus 10.8 0.615 3.00
Earth 15.0 1.00 2.96
Mars 22.8 1.88 2.98
Jupiter 77.8 11.9 3.01
Saturn 143 29.5 2.98
Uranus 287 84.0 2.98
Neptune 450 165 2.99
Pluto 590 248 2.99

 GM 
Below is a Graph of a3 against T2: a 3   2  T 2  0
y  axis  4  x axis no y intercept
slope

Kepler’s Third law can also be verified using Newton’s second law equation:

Fg  mg  Fc  mac

 mg  mac

M  r  M  r  2 T  
2 2
Mm v2 M v2
G 2 m G 2  G 2  G 2 
r r r r r r r r

M r 2  4 2 T 2  M 4 2
G   G  2
r2 r r3 T
 4 2  3
T2   r T  r T  a
2 3 2 3

 GM 

Page 111 of 186


Now comparing two small masses (m1 and m2) orbiting the same large one (M) by taking the ratio of
the above Equation:

 4 2  3  4 2  3
T12    r1 and T2
2
  GM  r2
 GM   

T12  4 2   GM  r13 r13 T12 r13 T12 a13


      
T2 2  GM   4 2  r23 r23 T22 r23 T22 ra3

Example

Given that the Moon orbits Earth each 27.3 d and that it is an average distance of 3.84 × 10 8 m from
the center of Earth, calculate the period of an artificial satellite orbiting at an average altitude of 1500
km above Earth’s surface. Where the radius of the earth is 6380km.

Solution

T1  27.3d , r1  3.8 108 m, T2  ?, h  1500km, rE  6380km, r2  ?

 r2  h  rE  1500km  6380km  7880km

 T 2 r3 
Using Kepler’s 3rd Law  1 2  13  :
 T2 r2 

r23 r23 r23 2


 T2 2  T12  T2 2  T12  T2  T1
r13 r13 r13 2

32
r 
32
24.0h  7880km 
 T2  T1  2   T2  23.6d     1.93h
 r1  d  3.8 108 m 

Page 112 of 186


This is a reasonable period for a satellite in a fairly low orbit. It is interesting that any satellite at this
altitude will orbit in the same amount of time. This fact is related to the condition that the satellite’s
mass is small compared with that of Earth.

4.4 Satellites: Orbits and Energy


There are hundreds of artificial satellites orbiting the Earth together with thousands of pieces of
debris. The Moon’s orbit about Earth has fascinated humans from time immemorial. The orbits of
planets, asteroids, meteors, and comets about the Sun are no less interesting, and there are
unimaginable numbers of stars, galaxies, and other celestial objects orbiting one another and
interacting through gravity. All these motions are governed by gravitational force  Fg  . These orbits
have the following characteristics:

i. A small mass m orbits a much larger mass M .

This allows us to view the motion as if M were stationary without significant error. Mass m is
the satellite of M , if the orbit is gravitationally bound.

ii. The system is isolated from other masses.

This allows us to neglect any small effects due to outside masses.

As a satellite orbits Earth on its elliptical path, either its speed (or its orbital velocity v  GM r ),
which fixes its kinetic energy K , and its distance ( h ) from the center of Earth, which fixes its
gravitational potential energy U g , fluctuates with the fixed periods. However, the mechanical energy
E of the satellite remains constant. Since the satellite’s mass ( m ) is so much smaller than Earth’s
mass ( M ), we assign U g and E for the Earth-satellite system to the satellite alone. The potential
energy of the system is given by

GMm
Ug   (0)
r

with U g  0 for infinite separation, r is the radius of the orbit, assumed for the time being to be
circular, and M and m are the masses of Earth and the satellite, respectively.

To find the kinetic energy of a satellite in a circular orbit, we write Newton’s second law as
Gravitational force (Fg) = Centripetal force (Fc)

GMm v2
  m (1)
r2 r
Page 113 of 186
v2
where is the centripetal acceleration (ac) of the satellite. Then from Equation 1, the kinetic energy
r
is
1  GM  GMm 1
  U g 
1
K mv 2  m   (2)
2 2  r  2r 2
v

which shows that for a satellite in a circular orbit,

Ug
K  (3)
2

The total mechanical energy of the orbiting satellite is

1  GMm   GMm 
E  K Ug     
2 r   r 
1  GMm   GMm  GMm  1  GMm
E      1   (4)
2 r   r  r 2  2r

This tells us that for a satellite in a circular orbit, the total energy E is the negative of the kinetic
energy K:

 GMm 
 E    K (5)
 2r 
Eqn.2

 U  U
TAKE NOTE: E   K     g   g
 2  2

The negative sign in Equations 4 and 5 means that the satellite is bound to the earth by gravity. Just
like the Earth is bound to the Sun.

Kinetic energy for a satellite in an elliptical orbit of semi major axis a, we can substitute a for r in
Equation 4 above to find the mechanical energy as

GMm
E (6)
2a
Equation 6 tells us that the total energy of an orbiting satellite depends only on the semi major axis of
its orbit and not on its eccentricity e.
Page 114 of 186
Example
Four orbits with the same semi major axis shows how an orbit’s shape changes with its eccentricity
are shown in the Figure (a) below. The same satellite would have the same total mechanical energy E
in all the four orbits. Figure (b) shows the variation of K, Ug and E with orbit radius r for a satellite (m)
moving in a circular orbit about a massive (M) central body.

Figure(a) Figure (b)

Example
A playful astronaut releases a bowling ball, of mass m = 7.20kg, into circular orbit about Earth at an
altitude h of 350km.
a) What is the mechanical energy E of the ball in its orbit?
b) What is the mechanical energy Eo of the ball on the launch pad, at say Cape Canaveral?
From there to the orbit, what is the change E in the ball’s mechanical energy?

Solution
a) the key idea here is that we can get E from the orbital energy, given by Equation 4, if we first find
the orbital radius r. that radius is

r  R  h  6370km  350km  6.72  106 m

in which R is the radius of Earth.

Then from Equation 4, the mechanical energy is (M-mass of earth)

Page 115 of 186


GMm (6.67  1011 Nm 2 kg 2 )(5.98  1024 kg )(7.20kg )
E   2.14  108 J  214MJ
2r 2  6.72  106 m

b) the key idea is that, on the launch-pad, the ball is not in orbit and thus Equation 4 does not apply.
Instead we must find initial total energy E 0  K o  U o , where K o is the ball’s kinetic energy and
U o is the gravitational potential energy of the ball-Earth.
To find U o , we use Equation (0) to write (note: r  R  Rball  R , since the ball is not in orbit and
its radius is very much less than R):

GMm (6.67  1011 Nm 2 kg 2 )(5.98  10 24 kg )(7.20kg )


Uo     4.51 108 J  451MJ
R 6.37  106 m
Ug
The kinetic energy K o of the ball is due to the ball’s motion with Earth’s rotation (but K  ).
2
You can show that K o is less than 1MJ, which is negligible relative to U o . Thus the mechanical
energy of the ball on the launch pad is

Eo  K o  U o  0  451MJ  451MJ

The increase in the mechanical energy of the ball from launch pad to orbit is
E  E  Eo  (214MJ )  (451MJ )  237MJ

Page 116 of 186


Chapter 5: Properties of Matter
The properties of matter include any qualities that can be measured, such as an object's density,
pressure, mass, volume, length, temperature, etc. Recall: The four states of matter (solids, liquids,
gases and plasma).

5.1 Pressure
Pressure is defined as the average force per unit area at the particular region of fluid (liquid or gas) i.e.

F
P
A

where F is the normal force due to the liquid on the side of area A .

At a given point in a liquid, the pressure can act in any direction i.e. pressure is a scalar quantity. If the
pressure were not the same, there would be an unbalanced force on the fluid at that point and the
fluid would move. The logical basis for the statement that pressure is exerted equally in all directions,
then, is simply that otherwise the parts of the fluid would not be in equilibrium. Also pressure
increases with depth, h , below the liquid surface and with its density,  , so that pressure is directly
proportional to depth ( h ) and density (  ).

P  h  P  hg 

Where g is the acceleration of gravity.

SI for g is ms-2, h is in m and  is in kgm-3, then the pressure ( P ) is in Newton per meter squared
( Nm 2 ) or Pascal ( Pa ). The bar is a unit of pressure used in meteorology and by definition,
1 bar =105 Nm-2  105Pa
Pressure is often expressed in terms of that due to a height of mercury (Hg). One unit is the torr (after
Torricelli ( v   2gh 
1/2
)):  1torr  1mmHg  133.3Nm2  133.3Pa
TAKE NOTE

Page 117 of 186


From P  hg , it follows that the pressure in a liquid is the same at all points on the same horizontal
level in it. Thus a liquid filling the vessels shown below rises to the same height ( h ) in each section if
ABCD is on same horizontal level.

-------------------------------------------
- - - - - - - - - - - -- - - - - - - - - - - - - - - - - - - - -- - - - -
A B C D

Figure Pressure in a vessel is independent of the cross-section

Atmospheric Pressure
A barometer is an instrument for measuring the pressure of the atmosphere which is required in
weather forecasting. It consists of a vertical tube about a meter long containing mercury with a
vacuum at the closed top. The other end of the tube is below the surface of mercury contained in a
vessel B.

TAKE NOTE
When using a weather barometer (usually with a scale range between 28 and 31 inches
Hg) at a location above sea level, the reading must be corrected back to sea level.

760 mm = x
H

(A)
•C
(B)
Figure i Figure ii Figure iii

Page 118 of 186


The pressure on the surface of the mercury in B is atmospheric pressure A and since the pressure is
transmitted through the liquid, the atmospheric pressure supports the column of mercury in the tube.
Suppose the column is a vertical height h above the level of the mercury in B. Then if
h  760mm  0.76m and   13600kgm3 , we have

P  h  g  0.76  13600  9.8  1.013  105Nm-2

The pressure at the bottom of a column (C) of mercury 760mm high for a particular density and value
of g is known as standard pressure or one atmosphere (1atm). By definition,

atmospheric pressure  P   PA  PC  h  g

1 atmosphere=1.01325105 Nm 2  760mmHg =760 torr

Standard temperature and standard pressure is 0oC and 760mmHg respectively.

It should be noted that the pressure P at a place x below the surface of a liquid is given by

P  Hg

where H is the vertical distance of x below the surface.

In Figure ii above, a very long barometer tube is inclined at an angle of 60o to the vertical. The length
of mercury along the slanted side of the tube is x mm. If the atmospheric pressure is the same as in
Figure i, then the vertical height to the mercury surface is still 760mm.

So

H
o cos 60o   x cos 60o  H
60 x
x
H  x cos 60o  h  760mm
760mm 760mm
x   1520mm
cos 60 0.5

Page 119 of 186


TAKE NOTE: Characteristics of the Mercury Barometer

5.2 Archimedes’ Principle


An object immersed in a fluid experiences a buoyant (upthrust) force equal to the weight of the fluid
that it displaces. The buoyant force opposes gravity yet it is caused by gravity. See the Figure below.

Archimedes' principle of buoyancy: In


the Figure a 5-kg object immersed in
water is shown being acted upon by a
buoyant (upward) force of 2 kg, which
is equal to the weight of the water
displaced (equal to apparent weight
mg  5kg  g
loss) by the immersed object.
The buoyant force reduces the object's
apparent weight by 2 kg—that is,
from 5 kg to 3 kg.
m apparent weight  g   3kg  g
Page 120 of 186
Applications of the principle
The buoyant force of a fluid on an object depends on the weight W fluid  M fluid  g  of fluid displaced
and thus on the density of the fluid and the volume of the fluid displaced (since M  D V ).

In the case of a totally immersed object, the volume of fluid displaced is just equal to the volume of
the object, and therefore if the buoyant force  Fb  is measured and the fluid density known, the
density of the object can be readily calculated. From this known volume and its mass, the density of
the object may be found. It is not easy to measure directly the volume of irregularly shaped objects
with great accuracy, but Archimedes’ principle provides a way to find volume accurately, since only
balance measurements are needed. Mathematically:

Fb = Mass of fluid  M fluid   acceleration due to gravity (g )

 density of the fluid   fluid   volume of fluid displaced (Vfluid )  g

 Fb =   fluid  Vfluid  g  or   fluid  volume of submerged object (Vobj. )  g 

 Fb =  Vg = Weight of the fluid displaced by the body (W fluid )

If two substances have densities 1 and  2 , then the density of the second substance relative to the
first 2 1 .

mass m m m V 
Since density  , then  2  2 and 1  1   2 1    2  1 
volume V2 V1  V2 m1 

If one compares equal volumes of the two substances so that V2 =V1 , then the relative density

m V  m
 2 1   2  1   2
 V1 m1  m1

Above Equation shows that the relative density will equal the ratio of their masses or their weights.

The density of a substance relative to that of water is called the specific gravity of the substance, i.e.
the specific gravity of a substance is equal to the density of the substance divided by the density of
sustance
water ( ) or
 water
the specific gravity of a substance is equal to the weight of a certain volume of the substance
Wsustance
divided by the weight of an equal volume of water ( ).
Wof equal volume of water

Page 121 of 186


In accurate work it is necessary to specify the temperature at which the measurements are made.

Example
A chunk of copper suspended from a balance weighs 156.8g in air. When it is completely surrounded
by pure water at 20oC, the reading on the balance is 139.2g.
Calculate the specific gravity of copper.

Solution
Specific gravity = density of substance  density of water

The weight of copper in air = 156.8g

The apparent weight of copper in water = 139.2g

Therefore the buoyant force of the water = apparent weight loss of copper in water

= (156.8g - 139.2g)  g = 17.6g  g

By Archimedes’ principle, the weight of the displaced water = buoyant force of the water = 17.6g  g

weight of Copper
The specific gravity of the copper =
weight of equal volume of water

But the volume of the displaced water is equal to the volume of the completely immersed copper
which displaces it. Therefore

mCu  g 156.8g  g 156.8g


The specific gravity of copper =    8.91 .
mwater  g 17.6g  g 17.6g

Therefore the density of the copper in this sample is 8.91 times the density of pure water at 20 oC.

When the specific gravity of a substance is known, its density in any units can be calculated from the
known density of water.

Wherever the force of gravity acts on a fluid, the fluid exerts a buoyant force as a result of difference
in pressure at different levels.

 Every fish and submarine in the sea is buoyed up by a force equal to the weight of water
displaced. To remain submerged these objects must have a weight equal to or greater than the

Page 122 of 186


buoyant force. If they wish to move from one level to another, the balance between the force of
gravity and the buoyant force must be disturbed.

 Some fish can rise by expanding their bodies, thereby displacing more water.

Fish has density greater than that of water, thus experiences a greater force of gravity that pulls it
downward towards the bottom of an ocean. In order to swim, the fish needs to reduce its density
by increasing its volume while keeping the mass nearly the same. Fish achieve this by using swim
bladder. When a fish fills the swim bladder with oxygen, its volume increases without causing a
significant rise in its mass. This reduces the density of the fish. The fish experiences a greater force
of buoyancy/displaces more water that makes the fish move up in water. When a fish expels
oxygen from the swim bladder, its volume decreases which makes the fish to sink. Fish can remain
at a specific depth when the force of buoyancy is same as the force of gravity.
 Submarines are made to rise by decreasing their weight by forcing water out of their ballast tanks.
See the Figure below.

(c) floating

 The air of the earth’s atmosphere also exerts a buoyant force on all objects immersed in it e.g. hot
air balloons utilize the buoyancy of air. If a gas such as hydrogen or helium is used to inflate a
light-weight plastic envelope, the buoyancy force of the air can be considerably greater than the
weight of the hot balloon. Such hot balloons are used in making high altitude measurements of
various properties of the atmosphere.

Page 123 of 186


How hot air balloons fly: because the hot air inside the balloon is less dense than the air outside
of the balloon. The working principle is hot air rises and cold air sinks. Heating the air causes
molecules to move faster and far apart making the air less dense. So hot air balloons fly because the
hot air inside the balloon is less dense than the air outside of the balloon.

FBouyancy =  Air density outside of the balloon, air outside balloon – Air density inside balloon, air inside balloon 

 Volume of the balloon,Vballoon  acceleration due to gravity, g

Example

A 4 person balloon has a volume of 2,200 m3.

a) If the ambient temperature was 10 oC and the air inside the balloon was heated to 100oC.
Determine its buoyancy force.
b) If the same balloon was heated to 90 oC. Determine its buoyancy force.

Comment on the results.

Page 124 of 186


Solution

a) Temperature outside the balloon Tout  10 oC , temperature inside the balloon Tin  100 oC

FBouyancy =  Air density outside of the balloon, air outside balloon – Air density inside balloon, air inside balloon 

 Volume of the balloon,Vballoon  acceleration due to gravity, g

 Fb  (1.247 – 0.946) kg.m-3  2,200m3  g = [0.301  2,200]kg  g = 662.2 kg  g .

NOTE: kgm-3.m3.ms-2=kg.m.s-2=N

b) Temperature outside the balloon Tout  10 oC , temperature inside the balloon Tin  90 oC

 Fb  (1.247 – 0.972) kg.m-3  2,200m3  g = [0.275  2,200]kg  g = 600.5 kg  g

You can see that the hotter the air is inside the balloon (part a) the more buoyant the balloon will
generate. In this example by increasing the temperature of the air inside the balloon by 10 oC from
90oC to 100oC the balloon can lift an additional mass of (662.2 kg - 600.5 kg) = 61.7kg.

Example
A weather balloon has a volume of 0.5m3 when inflated. The mass of the envelope/balloon is 350g. If
the balloon is filled with helium, what weight of instrument can it carry a loft? The density of air is
1.29kgm-3 and the density of helium is 0.138 times the density of air.

Solution
Vballoon  0.5m3 , Wenvelope  350g  g , Winstrument  ?, air  1.29kgm3 , He   0.138 1.29kgm3 

By Archimedes’ principle, the buoyant force of the air = the weight of air displaced
= the weight of 0.5m3 of air
= (0.5m3  1.29kgm-3)  g = 0.65kg  g = 650g  g
Since the relative density of helium is  He   0.138 1.29kgm3   0.17802kgm3 , the weight of helium
in the balloon is
 WHe  mHe g  He VHe g  0.17802kgm-3  0.5m3  g  0.08901kg  g  89.01g  g  90g  g
Page 125 of 186
Therefore the weight of the envelope/balloon and helium = (350g+90g)  g = 440g  g

The excess of the buoyant force over the force of gravity = (650g - 440g)  g = 210g  g

Therefore if an unbalanced force of (10g  g ) is left available to produce upward acceleration, the
weight of the instrument load can be 200g.

Floating at the surface of liquids

According to Archimedes’ principle, if a liquid is displaced by a solid, the liquid exerts a buoyant force
on the solid. This is true for any fraction of the solid that displaces liquid e.g. when a boy is lifting a
stone out of water, he does not have to support the full weight of the stone until it is completely clear
of the water. When half of the stone’s volume is submerged, the buoyant force is half of the buoyant
force when it is completely submerged. Suppose that an object is placed in a liquid of density greater
than that of the object   fluid  object  . The buoyant force of the liquid will equal the force of gravity
before the solid is completely submerged. To go beyond this point of equilibrium, requires then an
extra downward force to be applied. If it is not, the object remains in equilibrium under the action of
balanced forces: it floats. When an object that will float is placed into a liquid, it will sink into the
liquid until the weight that it displaces is equal to its own weight (Buoyant force = weight (i.e. mg)).
This relation, a direct consequence of Archimedes’ principle, is sometimes called the law of flotation.

Laws of Floatation
A body floats in a liquid if:
1. Density of the material of the body is less than or equal to the density of the liquid.
2. If density of material of body is equal to density of liquid, the body floats fully submerged in liquid
in neutral equilibrium.
3. When body floats in neutral equilibrium, the weight of the body is equal to the weight of displaced
liquid.
Page 126 of 186
4. The centre of gravity of the body and centre of gravity of the displaced liquid should be in one
vertical line (see the Figure above).

Example
A block of wood weighing 120g has a volume of 180cm3. What fraction of its volume would be
submerged when floating in alcohol of density 0.80gcm-3?

Solution
mblock  120 g , Vblock  180cm3 , alcohol  0.80 gcm3 , Valcohol displaced  Vblock submerged  ? , fraction of Vblock
submerged (f) = ?
Fb
Wooden block

Alcohol
Fg

Archimedes’ principle:
 Floating objects are in equilibrium:   Fy  0  Fb    Fg   0  Fb  Fg

 The fraction  f  of a floating object which is submerged is equal to the ratio of the density of the
object to the density of the fluid.

 Fb  Fg   mfluid displaced  g   mobject  g   fluid Vfluid displaced g  object Vobject g


 mobject submerged  g  Vobject submerged

Vfluid object
 Fraction of the volume of floating object which is submerged  f   
Vobject  fluid

Thus use density ratio (density of block divide by density of alcohol) or volume ratio (volume of
submerged block or volume of fluid displaced divide by volume of block), as shown in above Equation,
the latter is used.

Volume of alcohol displaced  Valcohol displaced  :


For the block to float, it must displace 120g mass of alcohol. The volume of 120g of alcohol

malcohol displaced 120g


Valcohol displaced    150cm3
alcohol 0.80gcm 3

Page 127 of 186


To displace 150cm3 of alcohol, the block will have to have 150cm3 of its volume submerged
( i.e. Vblock submerged  150cm3 ).

Vfluid Valcohol displaced 150cm3 5


The fraction submerged (f)    
Vobject Vblock 180cm3 6

TAKE NOTE
Archimedes’ principle provides the basis for the design of ships made of steel. For a steel vessel
to float it is only necessary to spread the steel around so that it can displace an amount of water
having a weight that exceeds the weight of the steel.

Example
A steel box is constructed to make a cube 10cm on a side, from material 0.20cm thick. What weight of
contents is possible before the box sinks in a liquid of specific gravity 1.2? The density of steel is
7.0gcm-3 and density of water is 1.0gcm-3.

Solution
L W=10 cm
steel sheet
W
L=10 cm

Thickness = 0.20cm
L W

The volume of the box is (10cm)3 = 1000cm3, specific gravity = 1.2, steel  7.0 gcm3 , water  1.0 gcm3 ,
liquid  ? , Wbox and contents fully submerged  ?, Wcontents  excess Fb  ? .

But specific gravity = density of substance  density of water

 Density of the liquid  liquid  = specific gravity  density of water   water 

And the maximum buoyant force ( Fb ,max )   mliquid displaced  g   liquid Vliquid displaced g
 mbox and contents fully submerged  g  Vbox submerged

 Fb ,max the liquid can provide = 1.2 1.0gcm-3   1000cm3  g  1200g  g

The weight  Wbox  of the box which has 6 sides, each 10cm square and 0.2cm thick, and which is made
of steel of density 7gcm-3 is
Page 128 of 186
Wbox   steel Vbox  g   steel  L×W×thickness per steel sheet  6 sheets   g
 

 7gcm3  10cm 10cm  0.2cm   6  g  840g  g

The excess of buoyant force over weight is

 excess Fb  Fb,max  Wbox  1200g  g  840g  g  360g  g

If material is added to the box, it will continue to float until the weight of the contents
exceeds  3.6 102 g   g .

Wcontents   3.6 102 g   g

HYDROMETERS

The relation between fluid density and floating/buoyancy provides basis for the construction and use
of hydrometers. Hydrometers usually consist of a hollow tube weighted at one end (filled with lead or
mercury) and having a graduated scale (to measure specific gravity = [density of substance  density
of water]) at the other end. See the Figures below.

Buoyancy bulb

(gravity bulb)

Figure i: Density salinometer used to Figure ii


determine salt content in water
densities in g/mL

Page 129 of 186


In a liquid, the weighted end ensures that the instrument floats upright. The depth to which it sinks
will depend on the density of the fluid. The higher the density of the liquid, the greater will be its
buoyant force per unit volume of the hydrometer immersed. When the scale is graduated using
liquids of known density (low-density liquids such as gasoline, alcohol and kerosene as well as high-
density ones such as acids, brine and milk), the hydrometer may be used to measure the density of
unknown liquids. A number of specialized hydrometers are used for the determination of specific
gravity (i.e. density of substance  density of water) of liquids:

 Lactometer for milk


 Alcoholmeter for wines and alcohols
 Saccharometer for sugar
 Urinometer for urine analysis
 Acidometer for acids
 Battery hydrometer for lead-acid battery
 Salinometer for liquids with salt content
 Thermohydrometer for the specific gravity of crude oil and petroleum products
etc.

Example
A hydrometer is used to measure the specific gravity of a liquid. A particular hydrometer is 40 cm in
length. When placed in pure water, 30 cm of the hydrometer is submerged and it reads 1.000.
a) What is the density of the hydrometer?
b) When placed in alcohol, 38 cm of the hydrometer is submerged beneath the alcohol. What is the
density of the alcohol?
c) What is the reading of the hydrometer when it is in the alcohol?

Solution
a) Free body diagram:

40cm Fb 30cm

pure water
W

Page 130 of 186


The hydrometer floats in water: F y  0  Fb   W   0  Fb  W

 Fb  W  mfluid displaced  g  mhydrometer g   fluidVsubmerged  g  mhydrometer g

  fluidVsubmerged g  hydrometer Vhydrometer g

  fluid  Ahsubmerged  g  hydrometer  Ahhydrometer  g ,

where A is the area and h is the height of the hydrometer

  fluid hsubmerged  hydrometer hhydrometer

 1000kg / m3   30cm   hydrometer  40cm 

 hydrometer  750kg / m3

b) Alcohol is less dense than pure water, the larger portion of the hydrometer will be submerged
in alcohol, e.g. 0.900, 1.000, 1.100 calibrations are submerged too (which are specific
gravities). See the free body diagram below.

40cm 10cm = (40cm – 30cm)

2cm = (40cm – 38cm)


30cm
40cm
38cm

alcohol

  fluid hsubmerged  hydrometer hhydrometer

 alcohol  38cm    750kg / m3   40cm 

 alcohol 
 750kg / m3   40cm  789kg / m3
38cm

Page 131 of 186


substance
c) specific gravity (SG) = density of substance  density of water 
 water

alcohol 789kg / m3
 SGalcohol    0.789
water 1000kg / m3

750kg / m3
SG for hydrometer   0.750
1000kg / m3

1000kg / m3
SG for water   1.000
1000kg / m3

alcohol

NOTE: The total density of the hydrometer is 750kg/m3, it can only meaure densities of liquids
that have density equal and greater than 750kg/m3. If the density of the liquid to be measured
is less than 750kg/m3, the whole hydrometer will sink in the fluid. Thus the highest mark for
this hydrometer is 0.75.

5.3 Fluids in motion

A stream / river flows slowly when it runs through open country and faster through narrow openings
or constrictions. This is due to the fact that water is practically an incompressible fluid, i.e. changes of
pressure cause practically no change in fluid density at various parts ( 1  2 ).

The Figure below shows a tube of water flowing steadily between X and Y, where X has a bigger cross-
sectional area A1 than the part Y of cross-sectional area A2. The streamlines of the flow represent the
directions of the velocities of the particles of the fluid and the flow is uniform or laminar.

l1 A2
v2
Y
A1 S
h2
v1 F2
X F1 Q l2
h1
R

Page 132 of 186


P
Assuming the liquid is incompressible (i.e. the density of the liquid doesn't change), then if it moves
from PQ to RS, the volume of liquid between P and R is equal to the volume between Q and S. This is
the continuity equation:

V1  V2 or

[the rate of change of mass  m1  = the rate of change of mass  m2  ].

Deriving the continuity equation

Fluid flow  Q  is the volume  V  of fluid that moves past a certain point per unit time  t  .

V
Q
t

Using the above Figure,

V1  A1l1  l  V2  A2l2  l 
Q1    A1  1   A1v1 and Q2    A2  2   A2 v 2
t t t t t t 

Fluid flow in section l1 must be equal to flow in section l2 , i.e. Q1  Q2 . If this doesn’t hold (i.e.
Q1  Q2 ) the fluid will pack up somewhere and become very compressed and will violate the
assumption that the liquids are incompressible.

 Q1  Q2

 A1v1  A2 v2 is the continuity equation (conservation of flow/law of


conservation of mass as applied to moving liquids)

l2 A1
Also VPR  VQS  A1l1  A2l2  
l1 A2

where l1 is PR and l 2 is QS.

Hence l 2 is greater than l1 ,  i.e. l2 l1  . Consequently the velocity  v 2  of the liquid at the narrow part Y
of the tube, where the streamlines are closer together, is greater than at the velocity  v1  at the wider

Page 133 of 186


part X where the streamlines are further apart,  i.e. v2  v1  . For the same reason, slow-running water
from a tap can be made into a fast jet by placing a finger over the tap to narrow the exit.

Example
A fire hydrant has a radius of 2cm and water leaves it with a velocity of 4m/s, a horse pipe is
connected to it and has an exit nozzle of radius 1cm. What will be the velocity of water as it exits the
nozzle?

Solution
Let velocity and radius for the hydrant be v1 and r1 respectively ; and velocity and radius for the nozzle
be v2 and r2 respectively . Using continuity equation  A1v1  A2 v 2  :

Example
In an adult, the aorta’s radius is normally appoximately 1.5cm, and blood moves throught it at an
average speed of 30cm/s.
If the typical capillary has a radius of 5 106 m , and blood passes through them with a velocity of
0.1cm/s, approximately how many capillaries are in the body?

Solution
raorta  1.5cm  r1 , vaorta  30cm / s  v1 , rcapillary  5 106 m  r2 , vcapillary  0.1cm / s  v 2 , N capillaries  N  ?

Using the continuity equation:

 A1v1  A2 v2

  r12 v1   r2 2  N  v2

  (1.5 102 m)2  3 101 m / s     5 106 m   N 1103 m / s 


2

Page 134 of 186


 (1.5 102 m)2  3 101 m / s 
N   2.7 109
  5 10 m  110 m / s 
6 2 3

BERNOULLI’S PRINCIPLE

Daniel Bernoulli obtained a relation between height  h  , pressure  P  and velocity  v  at different
parts of a moving incompressible fluid.

 
 static pressure dynamic pressure hydrostatic pressure 
  2 
The Bernoulli’s Equation: in pressure form   P  v   gh  constant 
2
 pressure energy kinetic potential energy along a streamline 

 energy

 conservation of energy 

 P v2 
in energy form     gh  constant 
 2 along a streamline 

 P v2 
in head form    h  constant 
  g 2g along a streamline 

Thus Bernoulli’s Equation is a statement of conservation of energy. It states that along the streamline
the sum of pressure energy, kinetic energy and potential energy remains a constant. It is only
applied along a streamline. See the Figure below for the horizontal flow of a fluid from point 1 to
point 2.

 

If viscosity is negligibly small, there are no frictional forces to overcome. Hence the work done by the
pressure difference per unit volume of a fluid flowing a long a pipe steadily is equal to the gain in
kinetic energy per unit volume plus the gain in potential energy per unit volume.
The work done by a pressure in moving a fluid through a distance = [force  distance moved] =
[pressure  area  distance moved] = [pressure  volume].

Page 135 of 186


W
 W  Fh  P  A h  P  V  P
V

At the beginning of the pipe in the Figure above where the pressure is P1, the work done per unit
volume on the fluid is P1 {i.e. P = (F/A) = [(W/l) /A] = [W/lA] = [W/V]}; at the other end, the work done
per unit volume is P2. Hence the net work done on the fluid per unit volume Wnet  [P1 - P2].
1
The kinetic energy per unit volume = mass per unit volume  velocity2
2

1m 2 1 m 1
 v  v2   v2
2V 2 m   2

where ρ is the density of the fluid.

Thus if v2 and v1 are the final and initial velocities respectively at the end and the beginning of the
1
pipe, the the kinetic energy gained per unit volume   (v 2 2  v12 ) .
2

Further, if h2 and h1 are the respective heights measured from a fixed level at the end and beginning
of the pipe, the potential energy gained per unit volume = mass per unit volume  g  h2  h1 

m m
 g  h2  h1   g  h  h    g  h2  h1 
V m   2 1

 potential energy gained per unit volume   g  h2  h1  .

Thus from the conservation of energy:

Energy per unit volume before = Energy per unit volume after

From Wnet per unit volume = kinetic energy gained per unit volume + potential energy gained per unit
volume

1
  P1  P2    (v 2 2  v12 )   g (h2  h1 )
2

1 1
 P1   v12   gh1  p2   v 2 2   gh2
2 2

Page 136 of 186


1
Therefore [ P   v 2   gh  constant ], where P is the pressure at any part and v is the
2
velocity at that part. This is the Bernoulli’s Equation, that is used to see how pressure changes as the
liquid flows in two different locations of different diameters. Hence, for streamline motion of an
incompressible non-viscous fluid, the sum of the pressure at any part plus the kinetic energy per unit
volume plus the potential energy per unit volume there is always a constant.
Bernoulli’s principle – Bernoulli’s equation at constant depth (h1  h2):

 1 1 
The Bernoulli’s Equation  P1   v12   gh1  p2   v2 2   gh2  becomes:
 2 2 

1 1
 P1   v12  P2   v 2 2 , proves the Bernoulli’s principle
2 2

For horizontal flow an increase in velocity must be accompanied by a decrease in pressure, for energy
to be conserved (refer to Bernoulli’s Equation), and this is known as Bernoulli’s principle. The principle
shows that at points in a moving fluid where the potential energy change gh is very small, or zero as
in flows through a horizontal pipe, the pressure is low where the velocity is high; conversely, the
pressure is high where the velocity is low. Re-arranging the above Equation:

1 1
 P1  P2   v 2 2   v12
2 2


 P1  P2   v2 2  v12  , show the change in pressure.
2

Bernoulli’s equation for static fluids (v1  v2  0):

 1 1 
The Bernoulli’s Equation  P1   v12   gh1  p2   v2 2   gh2  becomes:
 2 2 

 P1   gh1  p2   gh2

In this case pressure increases with height (h).

If we assume that the fluid is incompressible, then the mass flow rate at point 1 and point 2 must be
equal  i.e. m1  m2  . This gives as the continuity equation which is the statement of conservation of
mass.

Page 137 of 186


m1 m2  V1  V2  A1h1  A2 h2
 m1  m2      
t t t t t t

A1
  A1v1   A2 v2  A1v1  A2 v2  v 2  v1
A2

Since A1  A2  v2  v1 .

 A    
You can substitute this Equation  v 2  1 v1  into Bernoulli’s Equation  P1  P2   v2 2  v12 
 A2   2 
when h1  h2 :

   A1 
2

 P1  P2    v1   v12 
2   A2  

  A2 
 P1  P2  v12  1 2  1 , it shows that pressure decreases between point 1 and point 2.
2  A2 

See the summary below.

Example
As a numerical illustration, suppose the area of cross-section A1 of point 1 in the Figure below is 4cm2,
the area A2 of point 2 is 1cm2 and water flows past each section in laminar flow at the rate
of 400cm3 s 1 . Determine the pressure head.

Page 138 of 186


Solution
V
Q1  Q2   400cm3 s 1
t

volume per second 400cm3 s 1


At point 1: speed v1 of water=   100cms 1  1ms 1
area 4cm2
volume per second 400cm3 s 1
At point 2: speed v2 of water=   400cms 1  4ms 1
area 1cm2

TAKE NOTE: Confirmed the magnitudes for velocity above as shown below
V Al
 Q1  1  1 1  A1v1  400cm3 s 1
t t
V2 A2l2
 Q2    A2 v 2  400cm3 s 1
t t

The density of water   1000kgm3


Pressure head (h) =?

TAKE NOTE: pressure head (h) is the height of a liquid column that corresponds to a particular
pressure exerted by the liquid column on the base of its container. It may also be called static
pressure head or static head.

1 1
Using Bernoulli’s Equation: P1   v12   gh1  p2   v 2 2   gh2 , when h  h1  h2
2 2
  
then it becomes  P1  P2   v2 2  v12  . Let P be the pressure difference, the latter Equation becomes
 2 
 1 2 
 P  2   v2 -v1  .
2

1
 1000kgm 3   42  12  ms 1   7.5  103 kgm 1s 2  7.5 103 Nm 2
2
P
2

static pressure hydrostatic pressure


P 7.5 103 kgm1s 2
But P  hg   h   0.77m
pressure energy potential energy
 g 1000kgm3  9.8ms 2

The pressure head is thus equivalent to 0.77m of water.

Page 139 of 186


APPLICATIONS OF BERNOULLI’S PRINCIPLE
1. A suction effect is experienced by a person standing close to the platform at a station when a fast
train passes. The fast-moving air between the person and the train produces a decrease in pressure
and excess air pressure on the other side may push the pedestrian towards the train.
2. Filter pump. A filter pump has a narrow section in the middle, so that a jet of water from the tap
flows faster here. This causes a drop in pressure near it and air therefore flows in from the side tube
to which a vessel is connected. The air and water together are expelled through the bottom of the
filter pump.
3. Aerofoil lift. The curved shape of an aerofoil creates a faster flow of air over its top surface than the
lower one. This is shown by the closeness of the streamlines above the aerofoil compared with those
below. From Bernoulli’s principle, the pressure of the air below is greater than that above, and this
produces the lift on the aerofoil.

4. Flow of a liquid from wide tank.


Consider the Figure below. At the top X of the liquid in the tank, the pressure is atmospheric say P1 ,
the height measured from a fixed level such as the hole is h, and the kinetic energy is negligible if the
tank (i.e. v1 ) is wide so that the level falls very slowly. At the bottom Y near the hole, the pressure is
1
again P1 , the height is zero and the kinetic energy is  v e 2 where  is the density of the fluid and v e is
2
the velocity of emergence of the liquid.

where v1  0; h   h1  h2  ; if h2  0  h  h1; P2  P1; v 2  ve

Page 140 of 186


1 1
Using Bernoulli’s Equation: P1   v12   gh1  p2   v 2 2   gh2
2 2

1
Thus from Bernoulli’s principle: P1   gh  P1   v2 2  v2  2 gh
2

Thus the velocity of the emerging liquid ( v 2 ) is the same as that which would be obtained if it fell
through a height h and this is Torricelli’s theorem.

In practice the velocity of the emerging liquid ( v 2 ) is less than that given by 2 gh owing to viscous
forces and the lack of streamline flow.

Example
Water flows steadily a long a horizontal pipe at a volume rate of 8  10 2 m 3 s 1 . If the area of cross-
section of the pipe is 40cm2,
i. calculate the flow velocity of the water.
ii. Find the total pressure in the pipe if the static pressure in the horizontal pipe is 3.0  10 4 Pa ,
assuming the water is incompressible, non-viscous and its density is 1000kgm-3.
iii. What is the new flow velocity if the total pressure is 3.6  10 4 Pa .

Solution
V
Q   8  102 m3 s 1 , A  40cm2 , vwater  ? , Ptotal  ? , Pstatic pressure  3.0 104 Pa , water  1000kg / m3 ,
t
vnew  ? , Ptotal  3.6 104 Pa

Using Bernoulli’s Equation in form of pressure:

 
 static pressure dynamic pressure hydrostatic pressure 
  2 
 P  v   gh  constant 
2
 pressure energy kinetic potential energy along a streamline 

 energy

 conservation of energy 

volume per second Q 8 103 m3 s 1


i. Velocity of water  v water      2ms 1
area A 40 10 m4 2

ii. Since water is flowing the hydrostatic pressure is zero.

Page 141 of 186


Total pressure  Ptotal   static pressure + dynamic pressure

1000kg / m3   2m / s 
2
1
= Pstatic pressure   v2   3.0 104 Pa    3.2 104 Pa
2 2

iii. Total pressure  Ptotal   static pressure + dynamic pressure

 dynamic pressure = Total pressure  Ptotal   static pressure


1
  v new 2  3.6 104 Pa  3.0 104 Pa
2
1 0.6 104 Pa
  1000kg / m3  v 2  0.6 10 4 Pa v 3
 3.5ms 1
2 500kg / m

5.4 Elastic Properties of Matter

A perfect rigid body has constant distance between two particles (not true in practice), i.e. most
bodies get deformed under an applied force, and the body has a tendency to regain its original size
and shape when the force is removed. This property of the body that tends to regain its shape or size
when deforming forces are removed is called elasticity.
A perfectly elastic body retains its original shape/size very quickly, while if it regains slowly then it is
called a perfectly plastic body.
Solids tend to resist change of both shape and volume and hence they possess rigidity or shear
elasticity as well as volume elasticity.
Liquids on the other hand tend to resist change in volume and not shape (they possess only volume
elasticity).

STRESS  
When a body experiences a deforming external force, different particles in it are displaced, and they
try to occupy their original positions. This restoring force per unit area taking place inside the body is
called the stress. As long as there is no permanent change in shape or volume of the body, the
restoring force ( Fr ) is always equal to the applied force ( F ). See the Figure below.

Fr / A

Force( F )
Mathematically: Stress     Pressure  P 
Area( A)

Page 142 of 186


where A is the cross sectional area of the body.

STRAIN   
When an external force acts on a body, it displaces various particles and the body is said to be under
strain. Usually defined as the ration of change in length, volume or shape to the original length,
volume or shape respectively. See the Figure below.

L V change in shape
Mathematically: strain     or or
L V original shape
Strain is dimensionless.

YOUNG’S MODULUS (E) - MODULUS OF ELASTICITY


It is a measure of the ability of a material to withstand changes in length when under lengthwise
tension or compression. Young’s modulus is the ratio of stress to longitudinal strain within the elastic
limits. See the Figure below.

An

Mathematically:
The above metal bar has a cross-sectional area A and it is pulled by a force F at each end, the bar
stretches from its original length L0 to a new length Ln. Simultaneously the cross section area An
decreases.

F L Ln  L0
stress    and strain is     
A L L0

Page 143 of 186


 F/A FL0 F
Young’s modulus  E      L0 
  Ln  L0  L0  Ln  L0  A  Ln  L0  A

This is a specific form of Hooke’s law  F  k x  of elasticity. Because a graph of F against x is similar
to the graph of stress against strain, so k is a positive slope just as E . The units of Young’s modulus is
N/m2.

BULK MODULUS (B)


The bulk elastic properties of a material determine how much it will compress under a given amount
of external pressure. The ratio of the change in pressure to the fractional volume compression is
called the bulk modulus of the material or it is the ratio between stress and volumetric strain. See the
Figure below.

Deriving B using the ratio between stress   and volumetric strain   V  :


If a force is applied normally over a surface of a body in three mutually perpendicular directions and it
changes only in volume, then the strain caused here is volumetric strain measured by change in
volume  v  per unit volume v .

stress  F A F V
 Bulk modulus( B)    
volumetric strain  V ΔV V V  A

OR

Deriving B using the ratio of the change in pressure to the fractional volume compression:

change in pressure P P  V
 Bulk modulus( B)   
fractional volume ΔV V ΔV

Page 144 of 186


HOOKE’S LAW
According to the Hooke’s law, the stress   is proportional to the strain    (in the elastic region),
and the slope is Young’s modulus  E  .

stress  
Mathematically:       E   Young's modulus of elasticity (E )
strain  

where E is the coefficient of elasticity or Young’s modulus of elasticity (value depends on the nature
of the material).

Graphical representation for this relationship is as shown in the Figure below.

From the above graph, Hooke’s is only valid in the region below the elastic limit (the red line), beyond
which the body has un-proportional variation in strain and stress (where the change in stress leads to
a rapid change in strain).

5.5 Viscosity
Viscosity is a property by which a liquid opposes relative motion between its different layers (see the
Figure below). Liquids like kerosene, alcohol, water, etc., flow easily (have low viscosity) while others
like honey, tar, glycerin, etc., flow with difficulty and are called viscous.
Original position
dx

v

Page 145 of 186


Shear stress/Tangential stress  s or   is the tangential force (in the plane of the lamina) divided by
the area across which the force acts (the area vector is at right angles to the plane of the lamina), i.e.,
 s  Ft A .

Shear strain  or   is the distance moved in the direction of the force (dx) divided by the
perpendicular distance (dy) that separates the opposing forces producing the "twist", i.e
 tan    =dx dy  dx D  . Angle  is called shearing angle and it is the measure of shearing strain.
So shear strain develops due to shear stress. It is just the ratio of relative displacement  dx  of any
layer to its perpendicular distance  dy  from fixed layer.

Modulus of rigidity or shear modulus  G or S or  or   , (associated with solids)


Is the elastic coefficient when a shear force is applied resulting in lateral deformation. It is the measure of the
rigidity of the body, given by the ratio of shear stress to shear strain, the SI unit is Pascal (Pa).

shear stress  s F A Ft dy F dy F D
G    t   
shear strain  dx dy dxA dxA Adx

The shear modulus of wood is 6.2×108 Pa and of steel is 7.2×1010 Pa, it implies that steel is a lot more
rigid than wood, around 127 times more.

EXAMPLE
A block of 60 mm x 20 mm x 60 mm dimensions and unknown material is kept on a table (the rectangle face is
placed on the table), and it is under a shearing force of 0.245N. Calculate the shear modulus of the material if it
is displacement by 5 mm.

SOLUTION
Free body diagram:

5mm

F  0.245N

L  60mm

20mm

60mm
Page 146 of 186
shear modulus  G   ?

shear stress
G
shear strain

F 0.245 N 0.245 N 2450


Shear stress  s      Nm 2
A 60  20 10 m
6 2
0.0012m 2
12

x 5mm 1
Shear strain     
L 60mm 12

shear stress 2450 12 Nm2 2450


Shear modulus  G     12  2450 Nm2
shear strain 1 12 12

Coefficient of viscosity   , (associated with liquids)


Viscosity is defined as the degree up to which a fluid resists the flow under an applied force  Ft  , it is
calculated in terms of the coefficient of viscosity, it is measured by the tangential friction force ( f t ) acting per
 dv 
unit area divided by the velocity gradient   under conditions of streamline flow  note that Ft   ft  . The
 dy 
most simplistic formula for measuring viscosity is:

shear stress

rate of shear strain

s Ft A Ft Ft F dy
      t
d
  d  dx  d  dx 
A
dv Adv
  A  
dt dt  dy  dy  dt  dy

dv
 Ft   A
dy

For the simple two flat plates geometry, dv  v; dy  D , the above Equation becomes:

Page 147 of 186


v
 Ft   A
D

The force  Ft  depends directly on the area of the plates and the velocity gradient between the plates
(see the Figure below).

ft

An equal force  ft  will act in the opposite direction of Ft due to viscosity  Ft   ft  and it will depend
on the following:

i. For a given area (A), greater speeds (v) need lager forces (Ft): ft   v

ii. Force is proportional to the area of the plate, since A is directly proportional to the amount
of fluid being moved: ft  A

iii. Force is also inversely proportional to the perpendicular distance (D) between the top and
bottom plates: ft  1 D
The larger the D, the smaller is the Ft needed to achieve a given speed with a given contact
area. D is like a lever arm, and the greater the lever arm, the less force that is needed.

iv. The greater the fluid’s viscosity (  ), the greater the force (Ft) needed to move the fluid:
ft  

Combining the above four dependencies into an Equation we have:

1 1
ft    v   A   f t      v  A 
D D

v
 f t   A , which is in agreement with Ft   ft  ft   Ft .
D

where eta (η) is the coefficient of viscosity and it depends on the nature of the liquid.
Page 148 of 186
If the two layers are very close to each other, then

dv
f t   A
dD

where (dv/dD) is the velocity gradient.

If A = 1cm2 and dv/dD = 1, then ft    Ft   , which gives the definition of coefficient of viscosity
as the tangential force per unit area  Ft A  required to maintain a unit velocity gradient(dv/dD = 1).

If Ft = 1 then η = 1 and the unit is Poise.

TAKE NONE
Ft dy
Coefficient of viscosity    it is analogous to
A dv

F dy F dy
Shear modulus G      
A dx A dx

Example
An object of mass 1.5×101kg is hanging from one end of a steel wire. The wire without the mass has
an unstretched length of 0.50m. What is the resulting strain and elongation of the wire? The cross-
sectional area of the wire is 1.4×10−2cm2 and Young’s modulus for steel 20×1010Pa.

Solution
m  1.5 101 kg , Lo  0.5m ,   ? , L   L  Lo   ? , A  1.4 102 cm2 , and E  20 1010 Pa .

Free body diagram:


By Newton’s Third Law: T  mg  F  mg

Lo Stress and strain are linearly proportional:


L
F L
T        E  E ……. (Hooke's Law)
L A Lo

Thus strain on the wire from above Hooke’s Law:

mg L F L mg
 strain       
Lo EA Lo EA
Page 149 of 186

L mg
 
1.5 101 kg   9.8m / s 2  5.3 104
Lo EA  20 1010 Pa   1.4 102 cm 2 

mg
The elongation  L    Lo   5.3  104   0.5m  2.6  10 4 m
EA

SUMARRY

STOKES’ LAW (for laminar flow)


If a body falls through a fluid (liquid or gas) then it carries along with it a layer of the fluid in contact
and hence it will tend to produce some relative motion between the layers of the fluid. This relative
motion is opposed by forces of viscosity, and the opposing force increases as the velocity of the body
increases. If the body if small enough, then the opposing force becomes equal to the driving force that
produces the motion. At that instance then the body moves with constant velocity known as terminal
velocity. Consider a small steel sphere falling through a viscous medium (see the Figure below).

Page 150 of 186


Upthrust/buoyant force

To determine terminal velocity the net force MUST be zero, that is:

W  D  U   steel Vsteel g  6 rv+mfluid displaced Vfluid displaced g

 
 4 3  4 
  steel   r  g  6 rv+mfluid displaced   r 3  g
 if3spherical  3 
 

 6 v= r g   steei   fluid   terminal velocity  v  = r 2 g


4 2 2   steei   fluid 
3 9 

See the graph below illustrating the terminal velocity.

Then the opposing force  F  6 rv  is directly proportional to the velocity  v  of the sphere and
depends on
i. coefficient of viscosity   of the medium
ii. radius  r  of the sphere
iii. density of the medium   fluid 
Combining the three we have: F  k .v. r , where k is a constant taken to be 6π.
Page 151 of 186
SECTION II
SOUND AND VIBRATIONS

Chapter 6: Wave Motion

Introduction
Wave motion is a form of disturbance that travels through the medium (e.g. fluid, solid) due to the
repeated periodic motion of particles of the medium about their mean positions. This disturbance is
transferred from one particle to the next, e.g. dropping a stone into a pond of water, waves are
produced where the stone strikes the water. These waves travel outward, the particles of water
vibrate only up and down about their mean positions. See the Figure below.

Wave motion also refers to the transfer of energy from one point to the other point of a medium, e.g.
transfer of energy like sound, heat, light, x  rays,   rays , radio waves, etc., takes place in the form
of wave motions.

Classification of Wave Motion based on the medium of propagation/movement:

Page 152 of 186


Thus there are two types of waves:
a) Mechanical waves are waves created by the disturbance of a medium as a result of repeated
periodic motion of the particles of the medium, e.g. ripples on water surface, vibration of a
stretched string, sound wave, shock wave generated by a supersonic jet plane, seismic wave
generated during earth quake, etc. The energy is propagated as a result of the motion of the
particles of the medium, but medium remains at its previous position. Mechanical waves are
classified according to the direction of vibration of the medium particles with respect to the
direction of wave propagation as transverse waves and longitudinal waves, and energy transfer
as standing waves and progressive waves.

Transverse waves: particles of the medium vibrate about their mean positions in the direction
perpendicular to the direction of propagation, e.g. water waves, vibration of stretched string, all
electromagnetic waves (e.g. radio waves, microwaves, infrared waves, X-rays, gamma rays, seismic
S-waves etc.), etc. See the Figure below
wavelength   

Longitudinal waves: particles of the medium vibrate about their mean position in the direction of
the wave, e.g. sound waves, ultrasound waves, seismic P-waves, etc. See the Figure below.
wavelength
rarefaction

compression

Where rarefaction region: the wave particles are moving away from a common point where
pressure and density is at its minimum.
compression region: the wave particles are moving toward a common point where pressure and
density is at its maximum.

Standing/stationary waves are waves that remain confined to a region without any transfer of
energy and momentum, e.g. waves produced by stringed musical instruments. When the string is
plucked, pulses travel along the string in opposite directions.

Page 153 of 186


Progressive waves are waves that transfer energy and momentum between the particles of the
medium. That is the wave is moving away from its source, e.g. electromagnetic waves, dropping a
stone into a pond of water, etc. See Figure below.

b) Electromagnetic waves (non-mechanical waves) are created by repeated periodic fluctuations of


electric and magnetic fields of the environment. It does not need a medium for its propagation.
They are transverse in nature and travel in a vacuum with velocity of light, e.g. radio waves, visible
light, x  rays,   rays , infrared ray, microwave, etc.

Characteristics of wave motion


1. Is a disturbance produced in the medium by repeated periodic motion of the particles of the
medium.
2. Only the wave travels forward while particles of the medium vibrate about their mean
positions.
3. There is a regular phase difference between various particles of the medium. This means that
the particle ahead starts vibrating a little later than a particle just preceding it.
4. The velocity of the wave  v  is different from the velocity U  with which the particles of the
medium are vibrating about their mean positions. There difference being that the wave travels
with a uniform velocity while the velocity of the particles is different at different positions. The
velocity of the particle is maximum at the mean position and zero at the extreme position.

Page 154 of 186


EQUATION OF A SIMPLE HARMONIC WAVE (General Wave Equation)
Since wave motion involves vibration of particles about the mean position, they are also a SHM type
of wave motion. See the Figure below for a 1D dimensional sine wave.

wavelength ( )

To introduce the wave equation including time and position dependence, we consider the
displacement of a particle P in SHM given by

y  a sin t (1)

where, ω is the angular velocity and a is amplitude.

TAKE NOTE: waves are said to be in phase when waves travelling together are a complete
number of wavelegths apart e.g. A and A’, B and B’ or particcles of a wave train have equal
displacement and velocity, e.g. C and C’, D and D’ in the diagram below are in phase.

Suppose another particle A is at a distance x from the right of P and the wave is traveling with velocity
v in positive x direction (or from left to right), then the phase lag goes on increasing in this direction. If
 be the phase difference, then the displacement of the particle A will be given by

y  a sin(t   ) (2)

where φ is the phase difference between the particles P and A.

The path difference between two successive troughs or crests is λ, the time difference is T and the
phase difference is 2π.

Page 155 of 186


So a path difference of (λ) corresponds to a phase difference of 2π, thus, a path difference (X)
between P and A corresponds to the phase difference (2πX/λ) as shown below:

  2
X 

2 X 2 
 2 X       kx and  .
  x

where k = 2π/λ is called wave number/propagation constant of the wave motion.

Hence the phase difference φ between P and A is:

2x


A path difference (λ) corresponds to a time difference (T), therefore, a path difference (x) corresponds
to a time difference of (xT/λ) as shown below.

 T
X t

XT t
 t  XT  t  and T 
 X

So the angular frequency ω is

2 2 2 X 2 X 2 v
  2 f     
T  t X  t  t 

Putting the values of  and  in Equation (2) it becomes:

Page 156 of 186


2 2
y  a sin(t   )  a  sin( vt  x) (3) Or
Eqn.2
 

2
y  a  sin (vt  x) (4)

Equation (4) represents the general equation for a simple harmonic wave.

TAKE NOTE: k  2 /  is propagation constant or wave number or wave vector.

If the wave travels in opposite direction i.e. along negative X-axis (or from right to left), then equation
(4) becomes:

2
y  a  sin (vt  x) (5)

From Equations (4) and (5)

2
y  a sin (vt  x) (6a) OR

y  asin ( t  kx ) (6b)

This is the General Wave Equation for a traveling/progressive wave in positive X direction with
velocity v.

DIFFERENTIAL EQUATION OF WAVE EQUATION

Is derived by taking the partial derivative of the General Wave Equation, that is differentiating it with
respect to t , while holding x constant and vice versa.

So we start by differentiating the Equation (4) with respect to t while holding x which gives

d d  2 
 y  a  sin (vt  x) 
dt dt   
Eqn. 4

dy 2 av 2
  cos (vt  x) (a)
dt  
Page 157 of 186
Further differentiation of Equation (a) w.r.t. t gives

 2 v  2
2
d2y
    a sin (vt  x) (b)
dt 2    

Also differentiating the Equation (4) with respect to x while holding t to get the compression of the
wave (in x space), i.e.

d d  2 
y  a sin (vt  x) 
dx dx   
Eqn. 4

dy 2 a 2
  cos (vt  x) (c)
dx  

Further differentiation of Equation (c) w.r.t. x will give the compression in terms of distance (in x -
direction), i.e.

 
d  dy  d  2 a 2 
    cos (vt  x) 
dx  dx  dx   
 Eqn. c


 2  2
2
d2y
 2            a sin (vt  x)
dx    

 2  2
2
d2y
 2    a sin (vt  x) (d)
dx    

dy  2 a 2 
Comparing Equation (a):  v cos (vt  x)  and
dt    
dy  2 a 2   dy   2 a 2 
Equation (c):   cos (vt  x)        cos (vt  x)  , we see that
dx      dx     

dy  dy 
  v  
dt  dx 

Page 158 of 186


dy dy
  v (e)
dt dx

 
And comparing Equation (b): d y   v2     2  a sin 2 (vt  x)  and
2 2

     
2
dt

 
Equation (d): d y     2  a sin 2 (vt  x)  , we have
2 2

     
2
dx

d2y 2
2 d y
  v (f)
dt 2 dx 2

Equation (f) represents the differential equation of wave motion.

So the General wave motion Equation is:

d2y d2y
  K (g)
dt 2 dx 2

where K  v 2 or wave velocity  v   K .

d2y 2
2 d y d2y 1 d2y
Equation (f) can be re-written as  v   , which can be extended in
dt 2 dx 2 dx 2 v 2 dt 2
Eqn. f

three dimensional plane wave equation using any wave function like psi ( ψ ) as:

2ψ 2ψ 2ψ 1 2ψ


    . OR
x 2 y 2 z 2 v2 t 2

 2 2 2  1 2ψ 1 2ψ
  2 2 2  ψ  .   2
ψ  . 2 , where 2 is Laplacian Operator.
 x y z  v 2
t 2
v 2
t

Page 159 of 186


PARTICLE VELOCITY ( U ) AND WAVE VELOCITY ( v )

General Equation of Simple Harmonic wave is

2
y  a sin (vt  x) (1)

where v is the velocity of the wave and y is the displacement of the particle.

dy
Velocity of a particle is denoted by U and given by U  .
dt

So differentiating Equation (1) w.r.t. time ( t ) is:

d d  2 
 y  a sin (vt  x) 
dt dt   

d  2 
U   a sin (vt  x) 
dt  

2 av 2
U  cos (vt  x) (2)
 

 2 
Maximum particle velocity U max  occurs when cos (vt  x)   1 :
  

2 a
 U max  v (3)

It implies that Maximum particle velocity is (2πa/λ) times the wave velocity (v).

And particle acceleration of a wave ( f ) is given by:

2  
d2 d2  2   2 v   2 
f  2 y f  2  a sin  (vt  x)   f      a sin  (vt  x) 
dt dt
 y


4 2 v2
 f  y (4) OR
2
Page 160 of 186
d  2 av 2  2 v  2
2
d d dy   
f  U f   cos (vt  x)   f     a sin  (vt  x) 
dt dt dt dt       

4 2 v2
 f  y (4)
2

For maximum acceleration ( f max ) y  a

4 2 v 2
 f max   a (5)
2

The negative sign shows that acceleration of the particle is directed towards its mean
position/opposite to the direction of motion. NOTE: Newton’s 2nd law of motion F  mf .

The slope of the displacement curve  dy dx  is determined by differentiating Equation (1) with
respect to x :

 
d d  2  dy 2 a 2
 y   a sin (vt  x)    cos (vt  x) (6)
dx dx    dx  
 Eqn. 1 

where  dy dx  represents the slope of the displacement curve.

From Equation (2): U  dy  v  2 a cos 2 (vt  x)  and Equation (6):    dy   2 a cos 2 (vt  x)  :
dt     dx    

dy dy
U   v 
dt dx

dy dy
U   v (7)
dt dx

Therefore
particle velocity U  at any instant  wave velocity  v   slope of the displacement curve at that instant .

Page 161 of 186


DISTRIBUTION OF PARTICLE VELOCITY U  AND PRESSURE  P  IN A PROGRESSIVE WAVE

If a wave is progressive (i.e., new waves are continuously formed) then there is a continuous transfer
of energy in the direction of propagation of the wave.

2
The Equation for plane progressive wave: y  a sin (vt  x) (1)

dy 2 av 2
and particle velocity is: U   cos (vt  x) (2)
dt  

dy 2 a 2
strain or slope of the displacement curve in the medium is:  cos (vt  x) ,
dx  
if dy dx is positive, it represents the region of rarefaction. If dy dx is negative, it represents the
region of compression.

So modulus of elasticity ( K ) of the medium is:

change in presure dP


K 
volume strain dy dx

dy  dy 
 dP   K  K   (3)
dx  dx 

dy dy
which means if is positive, dP is negative, that is it is the region of rarefaction. If is negative,
dx dx
dP is positive, that is it is the region of compression.

Differentiating Equation (1) w.r.t. x :


 
d d  2  dy 2 a 2
 y  a sin (vt  x)    cos (vt  x) (4)
dx dx  dx  
 Eqn. 1

Substitute Equation (4) in (3):
  
 dy    2 a 2    2 Ka 2
 dP  K     K     cos (vt  x)    cos (vt  x) (5)
 dx       
 
Eqn. 3   Eqn. 4 

Page 162 of 186


The graphs for displacement ( y ), particle velocity ( U ), and change in pressure ( dP ), (Equations 1, 2
and 5 respectively) are shown in the Figures below. Where P0 is the normal pressure of the medium
when the wave is not propagating through it.

2
y  a  sin (vt  x) :

2 av 2
U cos (vt  x) :
 

 dy 
dP  K     :
 dx 

v 36000cm / s
v  
 100cm
  360 Hertz
1 1
T   0.0027 s
 360 s 1
Page 163 of 186
2 v x x t x
y  a sin (vt  x)  a sin 2 ( t  )  a sin 2 ( ft  )  a sin 2 (  )
    T 

 frequency  or f   3

But c=νλ  v=

INTERFERENCE

When two sound waves are moving along a straight line in a medium, then every particle of the
medium is simultaneously acted upon by both of the waves.
If the two waves arrive at a point in phase (two crests or two troughs) superimpose and the resultant
amplitude is equal to the sum of the respective waves (this is the principle of superposition).
If the waves arrive at a point when they are completely out of phase (a crest of one falls over the
trough of another, then the resultant amplitude is equal to the difference of the individual
amplitudes.
This implies that at points where the two waves meet in phase will give maximum amplitude (hence
maximum sound intensity) while where they meet out of phase gives minimum amplitude (minimum
sound intensity). These phenomenon is called interference. See the Figures below.

Combined (wave 1 & 2


)

Wave 1

Wave 2

(a) (b)

Figure (a) constructive and (b) destructive interference

Page 164 of 186


Below are two waves having different amplitudes a1 and a2:

OR

  90o   180o

Let the above two waves having amplitudes a1 and a2 and/ or out of phase (  ) (destructive
interference) be represented by the wave motion equations:

2  2 
y1  a1 sin (vt  x) and y2  a 2 sin  (vt  x   ) 
  

where  is the phase difference between the two waves after some time.

The resultant displacement ( Y ) will be (i.e. superposition of sinusoidal waves):

 2   2 
Y   y1  y2   a1 sin (vt  x)   a 2 sin (vt  x   ) 
     

 2    2 2 
 Y   y1  y2   a1 sin (vt  x)   a 2 sin   vt  x    
       

Using the trigonometric identity Sin (A + B) = Sin A cos B + sin B cos A, the above Equation becomes:

Page 165 of 186


  
 2    2 2  
 Y  a1 sin (vt  x)   a 2 sin   vt  x     
       
  A B  

 2    2 2 2 2 
 a1 sin (vt  x)   a 2 sin  vt  x  cos     sin    cos   vt  x    
               

 2   2 2 2 2 
 Y  a1 sin (vt  x)   a 2 sin  vt  x  cos     a 2 sin    cos   vt  x  
             

 2   2 2 2 2 
 Y  a1 sin (vt  x)   a 2 sin  vt  x  cos     a 2 cos   vt  x   sin   
             

2   2     2   2  
 Y  sin (vt  x) a1  a 2 cos      cos   vt  x   a 2 sin    
            

2 2 2
Letting [a1 + a2 cos  ]  A cos , and {a2 sin  }  A sin , where    , leads to:
  

2   2 
 Y  sin (vt  x)  A cos    cos   vt  x    A sin  
     

 2 2 
 Y  A sin (vt  x) cos   cos (vt  x)sin  
   

The above Equation is analogous to the trigonometric identity Sin A cos B + cos A sin B = Sin (A + B),
the above Equation becomes:

 
 2 2
 Y  A sin (vt  x) cos   cos (vt  x) sin  
   
 
B B
A A
sin(A+B)

 2 
Y  A sin  (vt  x)    which represents a simple harmonic wave
 

Page 166 of 186


2 2 2
where    , which is B   and A= (vt  x) .
  

This shows that the resultant wave Y has the same frequency but a different amplitude and phase
from the component wave trains.

TAKE NOTE:
If a1=a2, 1  2 ,1   2 , and 1  2 the resultant wave Y will be as shown in the Figures below:

Special cases (when a1 ≩ a2):


1. When   0o , A = a1 + a2 and hence tan   0 showing that the resultant is in phase with the
component
2. when   180o , A = a1 – a2

From 1 and 2 above, it implies that when the phase difference between two waves is zero the two
waves reinforce each other. This also implies that the resultant has the same period and its amplitude
is the sum of the amplitudes of the component waves. However, if the two waves have a phase
difference of 180o, they destroy each other and the resultant amplitude is the difference between the
amplitudes of the component waves.

Conditions for interference of sound waves


1. the two wave trains must move in the same direction.
2. The two sources must give waves of same frequency and amplitude so that the positions of
maxima and minima are distinct.
3. the two sources must be in phase, i.e. must be coherent source (are sources of light which
emit a light wave having the same frequency, wavelength and in the same phase or they have
a constant phase difference).

Page 167 of 186


TAKE NOTE:
Deriving the Equation for Y :

Solution: where l is 

STANDING WAVES
If two waves of the same amplitude traveling in the opposite directions superpose each other, they
generate a standing wave.

Formation of stationary waves


Below are the conditions for the formation of standing waves:
a) two waves travelling in opposite directions along the same line of travel and in the same plane,
b) the waves have the same speed,
c) the waves have the same frequency, and
d) the waves have the same approximate amplitude.

As a result of superposition a resultant wave is produced. Depending on the phase difference


between the waves, this resultant wave appears to move slowly to the right or to the left or disappear
completely. It is only when the phase difference is exactly zero, that is when the two waves are
exactly in phase, that 'standing/stationary waves' occur. See the waves below.
1. Two waves having the same amplitudes approach each other from opposite directions.

Page 168 of 186


2. The two waves below are 180o out of phase with each other, thus cancel out.

3. The phase difference between the two waves below narrows. The resultant grows but is not in
phase with either of the two waves.

4. The phase difference between the two waves is narrower still. The resultant is larger but is still
out of phase with the two waves

5. The phase difference between the two waves is now zero. The resultant has its maximum value
and is in phase with the two waves.

Properties of standing/stationary waves

The diagram shows how a standing wave (i.e. resultant of waves 1 & 2) moves up and down over time.

1. Separation of adjacent nodes is half a wavelength (λ/2).


2. Separation of adjacent antinodes is also λ/2
3. Separation of adjacent nodes and antinodes is λ/4 the maximum amplitude is 2a (twice that of
a single wave)

Page 169 of 186


4. a standing wave does not transfer energy (its two waves however, do transfer energy in their
respective directions)

The Figure below shows a string oscillating at its fundamental frequency/1st harmonic f1 which is the
lowest frequency for a standing wave to form:

1
1L  n 1
2

The string will vibrate in one segment (analogous to one anti-node).

These two waves are alternately in and out of phase and at regions of the waves being in phase, the
resultant is the sum of the waves with twice their amplitudes (anti-node points). The two waves
cancel out at points where they are out of phase forming nodes.

The frequency of the standing wave is the same as the frequency of the underlying traveling waves.
Examples are waves in a string that has fixed points at the ends. For the case as for the string with
fixed ends, the condition for standing wave is that the wave must be zero at the end points all the
times. Suppose the string has a length L, then the lowest possible standing wave to form is half
wavelength L  1 2 (fundamental mode - see the Figure above), the nest possible one is one
complete wavelength (known as the first overtone L  2 ) and so on, See the Figures below show
first overtone/2nd harmonic f 2 and second overtone/3rd harmonic f3 frequencies:

Page 170 of 186


The string will vibrate in two segments. The string will vibrate in three segments.

From the above three diagrams it can be seen that there is a pattern connecting the wavelength (λ) and
 2 L  L
the length of the string (L): 1  L,  , and 3  .
2 2 2 2 3

n L
For the (n-1)th overtone/ nth harmonic, the wavelength λn is given by:  .
2 n

This implies that for the modes (where waves form nodes at the ends) some number of half
wavelength exactly fits the length of the string. This can be expressed mathematically as:

2L
n 
n

where λ is the wavelength, n is the number multiple and L is the length.

Frequency theory

v
Using wave equation linking wave velocity, wavelength and frequency: v  f   f 

v
Hence the frequency of the nth harmonic ( fn ) is given by: f n  (i)
n

where λn is the wavelength of the nth harmonic and v is the velocity of the wave in either direction.

2L 1 n
But the wavelength λn is given by: n   
n n 2L
Page 171 of 186
v nv
Substituting for 1/λn into Equation (i) f n  :  fn  (ii)
n 2L

v
With n=1 , frequency of the 1st harmonic (the Fundamental) f1 is given by:  f1 
2L

Substituting for v/2L into equation (ii), we obtain the frequency of the nth harmonic in terms of the
Fundamental frequency:  f n  nf1

Thus proving that subsequent harmonics are all multiples of the Fundamental Frequency.

where f1 is fundamental frequency and the following ones fn as overtones.

All of these frequencies  f1 , f 2 , f3 ,.........., f n  can be changed by adjusting the tension T  in the
string. The greater the tension, the greater wave velocity (vw ) is and the higher the frequencies. This
observation is familiar to anyone who has ever observed a string instrument being tuned.

Effect of mass per unit length, length, and tension on frequency

The wave velocity of a transverse wave travelling in a stretched string is given by:

T
Experimentally it is shown that: v (iii)

where T is the tension in the string - Newtons (N), μ is the mass per unit length or linear density of the
string - (kgm-1).

nv 2 Lf n
From equation (ii) making v the subject: f n  v
2L n
Eqn.ii

T 2 Lf n T
Substituting for v from equation (iii): v   
 n 
Eqn.3

Page 172 of 186


n T
And making fn the subject:  fn  (iv)
2L 

1 T
From the Equation (iv) it can be confirmed that:  f1  (v)
2L 

So,

1
f1  ; T and  are constant . The law of length: Long strings make low frequencies and vice
L
versa. Thus the fundamental frequency is inversely proportional
to the restoring length of the string.

f1  T ; L and  are constant . The law of tension: tight strings make high frequencies and
vice versa. Thus the fundamental frequency is directly
proportional to the square root of the stretching force/tension.

1
f1  ; T and L are constant . The law of mass: thick, heavy strings make low frequencies

and vice versa. Thus the fundamental frequency is inversely
proportional to the square root of the mass per unit length.

The proportionalities are often termed the three Laws of transverse Vibration for Stretched Strings.

EXAMPLE

A wire of length 4.35m and mass 137g is under a tension of 125N. A standing wave has formed which
has seven nodes including the endpoints.

a) Which harmonic is it?


b) What is the frequency of this wave?
c) What is the fundamental frequency?
d) The maximum amplitude at the antinodes is 0.0075m. Write an equation for this standing wave.

Page 173 of 186


SOULTION

Free body diagram: The endpoints are closed, that is why there are nodes.

y1

y2

L  4.35m, M  137 g , T  125 N , N  7 nodes

a) Which harmonic is it?

Free body diagram

N=2 nodes, n   2  1 =1st harmonic

N=3 nodes, n   3  1 =2nd harmonic

N=4 nodes, n   4  1 =3rd harmonic

N=5 nodes, n   5  1 =4th harmonic

N nodes, n  (N-1) harmonic

 n  N  1   7  1  6 , so this is the 6th harmonic.

b) What is the frequency of this wave?

v 2L
Equations for a string fixed at both ends are: f n  n and n 
2L n

T T TL 125 N  4.35m
Wave velocity  v   v    630m / s
 M L M 0.137kg

Page 174 of 186


v TL M 630m / s
 fn  n n  f6  6  43.4 Hz
2L 2L 2  4.35m

c) What is the fundamental frequency?

Is obtained when a wave has two nodes (N=2): n  N 1  2 1  1

v 630m / s
fn  n  f1  1  7.24 Hz
2L 2  4.35m

v
Note: f n  n  nf1
2L

d) The maximum amplitude at the antinodes is 0.0075m. Write an equation for this standing wave.

Since a standing wave is formed, waves y1 and y2 are of the same amplitude, frequency/period
and travelling through a medium with same wave velocity but in opposite direction (are out of
phase by 180o) as shown in the figure above. According to superposition principle, the
displacement of the resultant wave Y  is:

Y  y1  y2  a1 sin t  kx   a 2 sin t  kx  , but a1 =a 2 =a


wave travelling along positive x -axis wave travelling along negative x -axis

 Y  a sin t  kx   a sin t  kx   a sin t  kx   sin t  kx 

 C+D   C-D 
From trigonometry identity: sin C  sin D  2sin   cos  ,
 2   2 

Page 175 of 186


 
 Y  a sin t  kx   sin t  kx  
 C D


  t  kx   t  kx    t  kx   t  kx   


 a  2 sin   cos  
  2   2  

 Y =2a sin t  .cos  kx   Y =2a sin t  .cos  kx 

 2 
 Y =2a cos  x  .sin  2 ft 
  
 2 
 Yn =2a n cos  x  .sin  2 f nt  is the displacement of resultant wave
 n 
Further more:

Y= 2a cos  kx  .sin t   A sin t   A sin  2 ft 


Amplitude of resultant wave

 Yn =An sin  2 f nt 

But Amax  at anti-node   A6  0.0075m  2a, f6  nf1  6  7.24Hz  43.44Hz, ,


2L 2  4.35m
n   6   1.45m :
n 6

 2   2 
 Yn =2a n cos  x  .sin  2 f nt   A n cos  x  .sin  2 f nt 
 n   n 

 Y6 =0.0075m  cos  4.33x   sin  272.94t 

TAKE NOTE:

  2  
Yn =  2a n cos  x   .sin  2 f nt  , resultant wave equation
  n  

Page 176 of 186


An  2a n cos  kn x  is the amplitude of resultant wave. This implies that the amplitude of the
stationary wave different at different points i.e. amplitude become zero at some points and
maximum at some other points.

Condition for maximum amplitude:

 2 
An  2a n cos  kn x   An  2a ncos  x
 n 

 2 
For Amax  2a n  cos  x 1
 n 

2
 x  n , n  0,1, 2,3,..........
n

2
 x  0 ,1 , 2 ,3 ,..........  x  0,  2,  ,3 2, 4 2,..........
n

Hence antinode occurs at x  0,  2,  ,3 2, 4 2,..........

Condition for minimum amplitude:

 2 
Amin  0  cos  x  0
 n 

Page 177 of 186


Review on characteristics of stationary wave:

1. The stationary wave are produced when two progressive waves of equal frequency and amplitude
travel in medium in opposite direction.
2. In the stationary wave, the disturbance or energy is not transmitted from particle to particle.
3. At nods, the particles of the medium are permanently at rest.
4. The particle at the anti-nodes vibrates with the maximum amplitude which is equal to twice the
amplitude of either waves.
5. The period of vibration of stationary wave is equal to that of either of wave.
6. The amplitude of particles on either side of an antinode gradually decreases to zero.

BEATS
When two waves of nearly same frequency travel along a straight line in the same direction, the
resultant displacement at a point varies in amplitude alternately. At any position, the amplitude of the
wave pulsates. This will be perceived as alternately soft and loud sound. See the Figure below.

The above Figure shows the production of beats (at i, iii, and v) and quiet spots at (ii and iv) by the
superposition of two waves 1 & 2 of slightly different frequencies but identical amplitudes. The
waves alternate in time between constructive interference and destructive interference, giving the
resulting wave a time-varying amplitude.

The frequency at which the amplitude pulsates is called the beat frequency ( fbeat or f ) and it is
mathematically given by:

fbeat  f 2  f1

Page 178 of 186


where f 2 ⋧ f1

The beat frequency is the number of beats produced per second, which is equal to the difference in
frequencies of the two interfering waves.

The phenomena of beats is used in tuning of musical instruments, e.g. to bring two flutes in tune,
musicians listen to beats and through trial and error, adjust one of the flutes to reduce the beat
frequency.

EXAMPLE
Two tuning forks A and B when sounded together produce 4 beats. If A is in unison with the 0.96 m
length of a sonometer wire under a tension, B is in unison with 0.97 m length of the same wire under
same tension. Calculate the frequencies of the forks.

SOLUTION
Watch this video link on how a sonometer works: https://www.youtube.com/watch?v=GTnPEtksTEc

L1  0.96 m; L2  0.97 m; fbeats  4s 1; f1  ?; f 2  ?


TAKE NOTE: since L1  L2  f1  f 2

fbeats  f1  f 2  4s 1  f1  f 2

 f 2  f1  4s 1

According to the 1st law/law of length of transverse vibrations (that the fundamental frequency is
1
inversely proportional to the restoring length of the string)  f1   f1L  constant :
L

Page 179 of 186


 f1 L1  f 2 L2

 f1  0.96m   f1  4s 1   0.97 m

 0.96mf1  0.97 mf1  3.88ms 1

 f1  0.97  0.96  m  3.88ms 1

 f1  3.88ms 1 / 0.01m  388 Hz

 f2   388  4  s -1  384 Hz

The frequency of the fork A is f1  388 Hz ,


The frequency of the fork B is f 2  384 Hz.

TAKE NOTE:
1) The number of beats per second equals the difference in frequency between the two
interfering waves.
2) The beat frequency is equal to the absolute value of the difference in frequency of the two
waves.
3) Sound is a pressure wave that consists of compressions and rarefactions.
4) The interference of sound waves causes the particles of the medium to behave in a
manner that reflects the net effect of the two individual waves upon the particles.
5) The beat frequency refers to the rate at which the volume is heard to be oscillating from
high to low volume.
6) The beat frequency is always equal to the difference in frequency of the two notes that
interfere to produce the beats.
7) When two waves of similar frequencies interfere, the result is a beat frequency.
8) A beat frequency is a pulsing sound that goes up and down in loudness.
9) As the two waves go in and out of phase, the varying constructive and destructive
interference makes the wave grow and shrink in amplitude. For sound waves this produces
a beating sound.
10) Second order beats occur between the two notes of a mistuned octave, and binaural beats
involve beating between tones presented separately to the two ears, so that they do not
mix physically.

Page 180 of 186


DOPPLER EFFECT OF SOUND
This is the apparent change in frequency due to relative motion between the source and the observer.
This originates from the reasoning that the standard speed of sound in air found to be 331m/s is
measured in a reference from at rest in air. However if the frame of reference is moving in air, the
speed will be larger or smaller, depending on the direction of motion of the reference frame. That is,
an observer will detect a higher frequency when approaching the source and a lower one when
receding. There are several cases where this occurs and we will explore each at a time.

Case 1: observer moving towards stationary source


Let the frequency of a source be f and the observer moving at velocity vo and the velocity of sound be
v. The observer encounters more wave fronts per second than when stationary.

In general the relationship between frequency, speed and wavelength of a wave is given by
f v 

In the frame of reference of the observer, the apparent frequency will be


f '  v' 
Where v’ is the velocity of the sound received by the observer. The wavelengths in the two equations
above are same since the distance between crests does not depend on the frame of reference. Hence

f ' v'

f v
The speed of sound will then be v’ = v + vo and therefore the apparent frequency heard by the
observer will be

 v+vo 
f ' f  
 v 
In general
 v 
f '  f 1  o 
 v

where + is for approaching observer and negative for receding observer.

Page 181 of 186


Case 2: Source moving and observer stationary
Apparent frequency is given by

 1 
f ' f  
 1  vs v 

Where + is for approaching source and – for receding source.

Other cases are governed in the following general equation:

 v±vo 
f ' f  
 v±vs 

where vo is velocity of observer, vs is velocity of source and v is velocity of sound.

Page 182 of 186


 v±vo 
The following conditions apply on the above general equation f '  f  :
 v±vs 
1. if observer is moving towards a stationary source, then add vo to v
2. if observer is moving away from source, subtract vo from v
3. if source is moving towards the observer, subtract vs from v
4. if source is moving away from observer, add vs to v

Exercise: Suppose both are moving in either direction, what would be the relationship?

Page 183 of 186


Page 184 of 186
Page 185 of 186
SECTION III

THERMAL PHYSICS - THERMAL PROPERTIES OF MATERIALS


Heat
Internal energy and temperature, phase changes of pure substance, isothermal and isobaric
compressibility of gases, liquids and solids.

Heat transfer
Conduction, convection and radiation.
Kinetic theory of gases, perfect gas equation, intermolecular forces, specific heats and equipartition of
energy.
(move to module page 53 by Z. Birech (birech@uonbi.ac.ke) Department of Physics, University of Nairobi_
PHYSICS_1A- LECTURER-COURSE NOTES-SECTIONS I-II-III)

Page 186 of 186

You might also like